Login

Welcome, Guest. Please login or register.

April 19, 2024, 10:44:00 pm

Author Topic: English Advanced Essay Marking (Modules Only)  (Read 602635 times)

0 Members and 2 Guests are viewing this topic.

literally lauren

  • Administrator
  • Part of the furniture
  • *****
  • Posts: 1699
  • Resident English/Lit Nerd
Re: English Advanced Essay Marking (Modules Only)
« Reply #60 on: January 19, 2016, 07:18:30 pm »
Well bangali beat me to it, and I'm just a VCE-er too, but my feedback has more to do with the structure of your information rather than the strength of your argument, so this might be helpful as well.

Comments in the attached doc. but you might have to open the comment pane to see them all (click the REVIEW tab and then click 'Reviewing Pane' under 'Tracking')

Again, please post here or let us know if you have any questions :)

aqsarana_

  • Adventurer
  • *
  • Posts: 9
  • School: Australian Islamic College of Sydney
  • School Grad Year: 2016
Re: English Advanced Essay Marking (Modules Only)
« Reply #61 on: January 20, 2016, 12:04:16 am »
Hi, I have copy pasted my discovery essay (Go Back to Where You Came From) for a check. I would really appreciate it if you can go through it and give me as much feedback as you can so i can improve on it. Thanks.

Is human driven by the need to discover? Discovery is to explore the unseen world and experience a world they may not have previously seen. It is through such discoveries that some individuals face ramifications which allow them to change perspectives of themselves and the world. This validates the statement that we can transform our understanding of ourselves and the world by travelling to new places or viewing a familiar place through new eyes. Discoveries can allow us to live by new values, however some choose not to. Discoveries enable new understandings and renewed perceptions of ourselves and the world. Go Back To Where You Came From (Go Back), a TV documentary series first aired in 2011 on SBS, explores the idea of travelling to places which contributes to new values and renewed perceptions inherited by some participants. Invictus, a poem by William Ernest Henley, published in 1998, affirms how seeing the world through new eyes approves for new values and understandings. Similarly, Sean Penn’s 2007 film Into The Wild, displays that travelling to a new place does bring about new values and renewed perceptions. All these texts greatly support the statement and let the characters, participants and viewers/readers to understand ourselves and the world better.
New values are not necessarily created by individuals when they go through a process of discovery. In Go Back, there are 6 participants with different perspectives and values. Even after hard experiences, some participants only change slightly and keep the same mentality. After the leaky boat experience, when they get onto the safe boat, Darren is questioned a lot. Darren started off with the attitude of how he doesn’t agree with people who put themselves on the boat when “they’re already safe”. While being questioned on the safe boat, he is juxtaposed with Gleny standing beside him, perhaps because they are participants of extremely different perspectives. With a medium camera shot, we see the expressions of Gleny when Darren says “I don’t feel empathy for people who willingly put their lives at risk when they are already safe”. This is ironic because Darren does not understand that they are unsafe and getting onto a boat is their last resort to stay safe.  During the raid in Malaysia (Episode 2), Raquel is seen as keeping her original values. The medium shot of her while saying “They should be doing this in Australia…” reflects her as being ignorant and confused as she says it without facing the camera but instead is looking at the raid. This illustrates how she is very stubborn on her viewpoint and instead of looking at it from a different perspective after discovering, she continues to look at it negatively. Overall, this shows that travelling to new places does not always transform our understandings of the world, shown through Raquel and Darren.
Hardships that an individual goes through allow them to discover new values. In Invictus, the writer writes about his own experience.  After he was diagnosed with a disease, he started to look at the world through new eyes which enabled him to discover himself and new values. In comparison to Go Back, some characters are not able to change their values and decide to stick to their original values. However, in Invictus, the poet is able to change his values and writes from a new perspective after being put into hardships. The use of simile and alliteration in “Black as the pit from pole to pole” (Line 2 – Stanza 1), gives the allusion of how severe his experience was. It contrasts the darkness of his suffering to the blackness of a hellish pit stretching from pole to pole. The severity of his hardship allowed him to understand the world and himself better. The visceral imagery used in “I have not winced nor cried aloud” (Line 6 – Stanza 2) describes that even though he has experienced pain, he will not show it remain strong. “I am the master of my fate: I am the captain of my soul” (Line 15 and Line 16 – Stanza 4) uses first person and anaphora, emphasising on the fact that the poem is his story and his response to his discovery. His discovery allowed him to make new values that assisted him to live his life. These quotes show how seeing the world with new eyes allows the individual to create new values and simultaneously gain a better understanding of ourselves and the world.
Discoveries can encompass new values and morals. In Into The Wild, before he sets off to his physical discovery, he discovers values that assist him in his physical discovery to Alaska. When compared to Go Back, they both have characters that change their values when they experience difficulties. In Invictus, the character also significantly changes his understanding of the world after going through discovery and this concept is discovered in the film Into the Wild too. In the flashback where the family is eating, the mother says to Christopher that they will gift him a new car. He replies with “I don’t need a new car. I don’t want a new car” and as he is replying, the camera focuses on his mother with a close up camera shot. Her expression illustrates the difference in values of his mother and himself. When Alex is on his journey to Alaska, he is sitting down and writing about his life in his diary. While he is writing, there is a medium camera shot of Alex playing with horses while the text: My days were more exciting when I was penniless” moves horizontally across the screen. This scene shows through his physical discovery he realised that he prefers life without money as money isn’t important for survival. 
Discoveries allow us to renew our perceptions of the world. After going through different experiences in Go Back, there are characters that are able to, at the end, renew their perceptions of the world. Their experiences allow them to forget about their existing perceptions and generate renewed perceptions. After leaving Deo’s family in Africa (Episode 3), Raquel changes her views and mentions that she doesn’t like the term “black people anymore…They’re not black people, they’re African people…” This is shown through a close shot of her face with slow, expressive music. This music makes the atmosphere look real and meaningful as it displays that her process of discovery allowed her to change and renew her opinions. Furthermore, in episode 2, when Adam is working with the Chins, he decides to ask how much the workers are paid. The medium shot shows his emotions as well as showing the background of the working conditions which represents hard work and dedication. The music also becomes tense and slow to create a hardworking atmosphere. With all this, we can see that Go Back does explore travelling to new places renews our perceptions and leads to a better understanding of the world and ourselves.
Discoveries enable new understandings of ourselves and the world. In Invictus, through the writer’s experience of discovery, he is able to change his perception on life. Similar to Go Back, the characters do experience renewed perceptions and this is because of the hardships they face. Lines 13 and 14, “It matters not how strait the gate, How charged with punishments the scroll,” contains a direct biblical allusion: “Strait is the gate, and narrow is the way, which leadeth unto life, and few there be that find it” (Matthew 7:13-14). This shows that he was able to discover spiritually which allowed him to gain a better understanding of the world and to renew his perceptions. In stanza 2, line 8, the use of visceral imagery and alliteration “my head is bloody but unbowed” shows that even though the poet experienced such hardships, he remained strong and pulled through it. He was able to hold his head high and this allowed him to see a new perspective of himself. These quotes and techniques show us how being able to renew our perspectives, we can understand ourselves and the world better. 
An individual’s perceptions are renewed when they discover. Into the Wild explores how different perceptions are created when a person is discovering and this is seen through the character of Christopher/Alex. This film can be associated with both Go Back and Invictus as they explore the same concept and comprise of characters that renew their perceptions after discovering. While Alex is sitting down and reading, there is a flashback of his past where his dad is beating up his mum. After the flashback, the camera focuses on Alex’s face as he has tears after reminiscing about that incident. Towards the end of the film, where his values change again, we see a hand-held camera along with a close up camera shot moving across Alex’s face and him writing: Happiness is only real when shared’. At the end of his discovery he realises that he was wrong when he thought life could be lived without family/friends. He wishes he could go back to his family to live again, however it is too late. This demonstrates to us his renewed perceptions because first he thought family and friends weren’t important, however through his discovery he realised he was wrong.
In conclusion, all three texts do explore characters that face ramifications and challenges by travelling to new places and in some cases just seeing the world with new eyes, which allow them to change their perspectives of themselves and their worlds. Go Back and Into the wild are about characters physically travelling to new places to discover, where as in Invictus, the poet sees the world with new eyes which enables him to renew his perspectives. The film Into The Wild has one main character that discovers and is able to change perspectives and values, whereas in Go Back, there are six participants that set off on a journey and experience differences in values at different stages. All three texts greatly show how our understanding of ourselves and the world may change when we travel to new places or view a familiar place through new eyes.   

literally lauren

  • Administrator
  • Part of the furniture
  • *****
  • Posts: 1699
  • Resident English/Lit Nerd
Re: English Advanced Essay Marking (Modules Only)
« Reply #62 on: January 22, 2016, 11:37:36 am »
Is human because you're talking about many/all people, you need the plural 'humans' rather than the singular 'human,' so this should be: 'Are humans...' driven by the need to discover? Discovery is to explore the unseen world and experience a world that may not previously have been seen. It is through such discoveries that some individuals face ramifications which allow them to change perspectives of themselves and the world. This validates the statement that we can transform our understanding of ourselves and the world by travelling to new places or viewing a familiar place through new eyes. Good expanding of what it means to discover something. I like that you're including 'seeing old things with new eyes' and not just 'seeing new things.' Discoveries can allow us to live by new values, however some choose not to. This is a bit brief - if you want to use this as a key point or argument, then maybe spend an extra sentence talking about it so the assessor knows what you mean. Discoveries enable new understandings and renewed perceptions of ourselves and the world you've said this already in a previous sentence. Go Back To Where You Came From (Go Back), a TV documentary series first aired in 2011 on SBS, explores the idea of travelling to places which contributes to new values and renewed perceptions inherited by some participants. A linking word would be good here. eg. 'Likewise' or 'Furthermore... 'Invictus, a poem by William Ernest Henley, published in 1998, affirms how seeing the world through new eyes approves Word Choice. This doesn't really fit here. Perhaps use 'facilitates' or 'creates' new values and understandings. Similarly, Sean Penn’s 2007 film Into The Wild, displays reveals that travelling to a new place does bring about new values and renewed perceptions This point seems a little simplistic. We already know that the poem says something about discovery leading to broader perceptions, but your introduction should be exploring these ideas in more detail. All these texts greatly support the statement and let the characters, participants and viewers/readers just pick one. 'Audience' is probably best. Don't use slashes (/) in your writing as they're seen as informal to understand ourselves and the world better.

New values are not necessarily created by individuals when they go through a process of discovery. Good! :) Topic sentence is based on an idea or concept.In Go Back remember to put the title in 'single quotation marks', there are 6 participants with different perspectives and values. Be careful here; you've started off well, but going from a really big idea about the process of discovery and then jumping into an example too soon can feel a bit sudden. (e.g. 'Often our perspectives are shaped by our life experiences. For example, in 'Go Back,' we see...' <-- notice how the sentences don't really connect very well?) Instead, try to gradually go from the idea to the example over two or three sentences if needed. This will let you explore the idea more fully too. Even after hard experiences, some participants only change slightly and keep the same mentality. After the leaky boat experience, when they get onto the safe boat, Darren is questioned a lot You're keeping your sentences nice and clear here, which is excellent, but I'm not too sure what you're referring to here. I'm guessing it's that Darren was interviewed, but you haven't really made that clear (i.e. who is he being questioned by? What are the questions?). Darren started off with the attitude of how he doesn’t agree with people who put themselves on the boat when “they’re already safe”. Good quote integration! While being questioned on the safe boat, he is juxtaposed with Gleny standing beside him, perhaps because they are participants of extremely different perspectives. With a medium camera shot, we see the expressions of Gleny when Darren says “I don’t feel empathy for people who willingly put their lives at risk when they are already safe”. This is ironic because Darren does not understand that they are unsafe and getting onto a boat is their last resort to stay safe. Why is Gleny important here? You bring him up but don't really explain the significance of him. How is Gleny different to Darren? And, more importantly, what does this example say about the prompt, and about discovery? During the raid in Malaysia (Episode 2) I don't think you have to specify this, or if you do, it should be part of the sentence, not in brackets (i.e. 'During the raid in Malaysia in Episode Two, Raquel is seen...') but check with your teacher to be sure, Raquel is seen as keeping her original values. The medium shot of her while saying “They should be doing this in Australia…” reflects her as being ignorant and confused as she says it without facing the camera but instead is looking at the raid. This illustrates how she is very stubborn on her viewpoint and instead of looking at it from a different perspective after discovering, she continues to look at it negatively. Overall, this shows that travelling to new places does not always transform our understandings of the world, shown through Raquel and Darren.
Try to link ^this discussion to the next one in the Topic Sentence below. How are these two ideas related? Start with a word like 'Similarly...'/'Likewise...' or 'Contrarily...'/'On the other hand...' and make a connection from there.
Hardships that an individual goes through allow them to discover new values. So why were the people in  your previous paragraphs unable to shift their world views? What's different in this case? In Invictus, the writer writes about his own experience.  After he was diagnosed with a disease, he started to look at the world through new eyes which enabled him to discover himself and new values. In comparison to Go Back, some characters are not able to change their values and decide to stick to their original values. Rather than going from Go Back in the previous paragraph, to a brief mention of Invictus, to Go Back, to Invictus again, try and minimise the amount of switching that you do. Conclude your Go Back paragraph, and then, at the start of the next one, make a strong link between the two texts and then just start analysing Invictus; there's no reason to go back and forth between them in this case. However, in Invictus, the poet is able to change his values and writes from a new perspective after being put into this is a bit informal; maybe say 'after experiencing...' hardships. The use of simile and alliteration in “Black as the pit from pole to pole” (Line 2 – Stanza 1) again, I don't think you have to cite the line and stanza number, but I'm not 100% sure, gives the allusion this expression is a bit odd; you can go for a simpler verb like 'represents' or 'shows' instead of how severe his experience was. It contrasts the darkness of his suffering to the blackness of a hellish pit stretching from pole to pole. AWESOME!!! I was just about to say that your previous sentence: 'this quote gives the allusion of how severe his experience was' wasn't really enough because you hadn't explained HOW that quote demonstrates your point, but you've done it right here!! :) Really good stuff, and something that a lot of essays miss out on - always make sure you're spelling out the link between your evidence and your ideas like this. The severity of his hardship allowed him to understand the world and himself better. The visceral imagery used in “I have not winced nor cried aloud” (Line 6 – Stanza 2) describes that even though he has experienced pain, he will not show it remain strong. <-- link these ideas? -->“I am the master of my fate: I am the captain of my soul” (Line 15 and Line 16 – Stanza 4) uses first person and anaphora, emphasising on the fact that the poem is his story and his response to his discovery. His discovery allowed him to make new values that assisted him to live his life. These quotes show how seeing the world with new eyes allows the individual to create new values and simultaneously gain a better understanding of ourselves and the world.
Rather than listing three points of evidence and then getting to the end of the paragraph to make the connection obvious, try to make links as you go! You don't want your paragraphs to read like a dot-point list of examples with some analysis just put one after another. Instead, you want to go from one point to the next. Think of it this way: if you bake a cake, you don't want the end result to be a pile of flour with some unbeaten eggs in it, covered in milk... that's not a cake  :P A cake involves blending the ingredients so you can get the right mixture. In the end, the assessors will still be able to point at it and say 'okay, I can see that there's chocolate, butter, and some milk in there' just like they'll be able to see your paragraph and think 'okay, I can see that you've used this quote and this example,' but there's still an overall structure. Sticking the ingredients together doesn't magically turn them into a cake; you have to combine them. So, when you're writing your paragraph, try to focus on building one example into the next. There must be a reason why you're going from one example to another one - maybe it reinforces your point, or maybe it demonstrates another facet of your argument. Whatever the reason is, mention it in your discussion so that your assessor can follow your logic :)

Discoveries can encompass new values and morals. Connect this with your previous discussion. In Into The Wild, before he sets off to his physical discovery, he who's 'he?' You haven't introduced the main character yet discovers values that assist him in his physical discovery to Alaska. When compared to Go Back, they both have characters that change their values when they experience difficulties. In Invictus, the character also significantly changes his understanding of the world after going through discovery and this concept is discovered explored in the film Into the Wild too. In the flashback where the family is eating, the mother says to Christopher that they will gift him a new car. He replies with “I don’t need a new car. I don’t want a new car” and as he is replying, the camera focuses on his mother with a close up camera shot. Her expression what is her expression? What emotions is she revealing? And how does this illustrate the fact that they have different values? illustrates the difference in values of his mother and himself. When Alex is on his journey to Alaska, he is sitting down and writing about his life in his diary. While he is writing, there is a medium camera shot of Alex playing with horses while the text: My days were more exciting when I was penniless” moves horizontally across the screen. This scene shows through his physical discovery he realised that he prefers life without money as money isn’t important for survival. Why is this discussion important? What does this tell us about the nature of discovery?

Discoveries allow us to renew our perceptions of the world. Again, you need a link between this sentence/idea and what you've just discussed in the previous paragraph. There is a connection, but because you haven't made it clear, you can't get marks for it. After going through different experiences in Go Back, there are characters that are able to, at the end, renew their perceptions of the world. Their experiences allow them to forget about their existing perceptions and generate renewed perceptions. slight repetition here; try to find some synonyms because this is a word that you'll be using a lot. After leaving Deo’s family in Africa (Episode 3), Raquel changes her views what were her views before, and what did she change to? You don't really explain this here, you just provide a quote and mentions that she doesn’t like the term “black people anymore…They’re not black people, they’re African people…” What does this quote mean for her character? What does it suggest she's changed from/into? This is shown through a close shot of her face with slow, expressive music. This music makes the atmosphere look real and meaningful as it displays that her process of discovery allowed her to change and renew her opinions. The music probably shouldn't be your focus here - concentrate on the words in that quote and what they signify. Furthermore, in episode 2, when Adam is working with the Chins, he decides to ask how much the workers are paid. The medium shot shows his emotions which emotions?? Calling it a 'medium shot' isn't very important, but talking about his facial expressions and emotions is crucial! as well as showing the background of the working conditions which represents hard work and dedication How do you know? What is it about the background that represents hard work? Describe what you see or hear in the show, and explain why these details reveal those ideas. The music also becomes tense and slow to create a hardworking atmosphere. With all this, we can see that Go Back does explore travelling to new places renews our perceptions and leads to a better understanding of the world and ourselves. Don't just restate the prompt; try to go a bit further in unpacking your ideas. Your examples shouldn't just be used to demonstrate a 'yes' or 'no' argument; they should allow you to explore more interesting points of view.

Discoveries enable new understandings of ourselves and the world. In Invictus, through the writer’s experience of discovery, he is able to change his perception on life. Similar to Go Back, the characters do experience renewed perceptions and this is because of the hardships they face. The previous sentence is about 'Go Back,' but this next one is about 'Invictus,' though you haven't made it clear that you've changed texts. Be careful when jumping between them.Lines 13 and 14, “It matters not how strait the gate, How charged with punishments the scroll,” contains a direct biblical allusion: “Strait is the gate, and narrow is the way, which leadeth unto life, and few there be that find it” (Matthew 7:13-14). This shows that he was able to discover spiritually which allowed him to gain a better understanding of the world and to renew his perceptions. Good. In stanza 2, line 8, the use of visceral imagery and alliteration “my head is bloody but unbowed” shows that even though the poet experienced such hardships, he remained strong and pulled through bit colloquial it. He was able to hold his head high and this allowed him to see a new perspective of himself. what was this new perspective, exactly? These quotes and techniques show us how being able to renew our perspectives, we can understand ourselves and the world better. Notice how this point could essentially be summarised as 'yes, the prompt is right.' Try to avoid this. Take things a step further and challenge the prompt a little bit if you want to push your ideas into better territory. More on this in the end comments.

An individual’s perceptions are renewed when they discover. Into the Wild explores how different perceptions are created when a person is discovering and this is seen through the character of Christopher/Alex. This film can be associated with both Go Back and Invictus as they explore the same concept and comprise of characters that renew their perceptions after discovering. While Alex is sitting down and reading, there is a flashback of his past where his dad is beating up his mum. After the flashback, the camera focuses on Alex’s face as he has tears after reminiscing about that incident. Towards the end of the film, where his values change again, we see a hand-held camera along with a close up camera shot moving across Alex’s face and him writing: Happiness is only real when shared’. At the end of his discovery he realises that he was wrong when he thought life could be lived without family/friends don't use slashes; say 'and' or 'or'. He wishes he could go back to his family to live again, however it is too late. This demonstrates to us his renewed perceptions because first he thought family and friends weren’t important, however through his discovery he realised he was wrong.

In conclusion, all three texts do explore characters that face ramifications and challenges by travelling to new places and in some cases just seeing the world with new eyes, which allow them to change their perspectives of themselves and their worlds. Go Back and Into the wild are about characters physically travelling to new places to discover, this isn't something you explore in much detail in your body paragraphs where as in Invictus, the poet sees the world with new eyes which enables him to renew his perspectives. The film Into The Wild has one main character that discovers and is able to change perspectives and values, whereas in Go Back, there are six participants that set off on a journey and experience differences in values at different stages. Try not to just re-list your evidence in your conclusion. Conclusions are about ideas, so you want to say something about discovery here, not just go back over your major exeamples. All three texts greatly show how our understanding of ourselves and the world may change when we travel to new places or view a familiar place through new eyes.

There's some really good stuff here, but a couple of things are holding you back.

The first big one is essay structure. Overall, it feels like you're switching between texts too often, and as such, your paragraphs often don't have a clear idea-based focus because you get to the end and just say 'therefore, discovery can lead to renewed perceptions' pretty much every time.

As a new kind of structure, try this:

Paragraph 1: Comparing Invictus and Into the Wild
Paragraph 2: Comparing Into the Wild and Go Back
Paragraph 3: Comparing Go Back and Invictus

Now, for each paragraph (e.g. Paragraph 2: Into the Wild + Go Back) think of a related idea that's present in both texts. For example, you state in your conclusion that they both explore the notion of changes in our physical place impacting our perception. So for this paragraph, you can start with a sentence like 'Often, changes in one's physical environment can mirror changes in our understanding and expectations.' Then you can spend the paragraph talking about and analysing both texts. Then, when you get to the end, you can take that idea and ask 'what does this say about the prompt?' The answer to that question will be your last sentence for that para.

So in other words, your paragraphs should aim to obey the following:
1. Outline a general (i.e. not text-specific) idea about discovery that's related to the prompt.
2. Zoom in to one of the texts
3. Analyse that text.
4. Find a connection between that first text and a second one.
5. Analyse that second text.
6. Zoom out a bit and talk about what that second text says about discovery.
7. Bring the texts together and talk about a point of similarity or difference.
8. Zoom all the way out to the prompt and link your paragraph's focus to the ideas raised by the prompt.


Secondly, whilst the quality of your analysis is pretty good, you need to connect one point of analysis to the next. It's like you're trying to build a lego tower; if you don't connect the bricks... then you just have a pile of bricks sitting around :P It's crucial that you make those connections really strong, and think about why you're going from one bit of evidence to the next.

This goes for your paragraphs too.At the moment, I could put your body paragraphs in any random order and nothing would be different. You want your discussion to feel like it's going from 1 --> 2 --> 3, not just 1 & 2 & 3 with no sense of progress and linking.

Lastly, but perhaps most importantly, make sure your essay has a contention or thesis statement. You have to build an 'argument' around an idea; you're not supposed to prove the prompt right - you're supposed to take that prompt as a starting point for your own thesis.

Think of it this way: if you got a prompt like 'Discovering who we are can take a long time.'
A mid-range essay would have a 'thesis' like: In order to discover who we are, we have to spend a long time working out our identities. See how it's just completely agreeing? There's no challenge, and there's no sense that the student writing this is taking the ideas further.
A high-range essay, though, might have a thesis like: 'Although we may think we know ourselves well, often the process of self-discovery is a long and arduous one, but this is ultimately a necessary experience if we want to truly understand ourselves.' There's waaaay more going on in this sentence now. We're talking about how the process is long and difficult, and how it's necessary even though it's a struggle.

If in doubt, go for the following formula:
'Although >challenge to the prompt that disagrees a bit< , ultimately >main argument that mostly agrees with the prompt, but looks at the reasons or consequences for this being true, too.< '

So, in summary; the stuff to work on:
- Linking between ideas
- Connections between paragraphs, especially in topic sentences
- Alternative essay structure that let you explore more ideas in more depth
- Stronger thesis statement based on, but not limited to the prompt

Best of luck!

Spencerr

  • Forum Regular
  • **
  • Posts: 98
  • Things will turn out better than expected.
Re: English Advanced Essay Marking (Modules Only)
« Reply #63 on: January 26, 2016, 09:13:52 pm »
   
Hey! I've recently 'discovered' this website and it's been sooo helpful. I was wondering if you guys could help check my essay on discovery. Any feedback would be invaluable! The essay is a generic one without a set question in mind.

The biggest struggle that I have right now is taking my expression and sophistication to the next level. What practice would you guys recommend so that I can do that? Thanks so much in advance.

Experiences of discoveries either challenge or affirm deeply held values, resulting in transformations and broadened perceptions. Within Shakespeare’s tragicomedy The Tempest, lies an exploration into the transformative power of discoveries supplemented by an examination into the relationship between one’s self and one’s world, enabling Shakespeare to renew perspectives on colonialism. However, the multi-faceted nature of discoveries espouses the differing ramifications imposed upon individuals, as Edgar Allen Poe’s psychological short story, The Tell Tale Heart, conversely depicts the limitations of discoveries in evoking change.  Nonetheless, Poe similarly creates fresh perspectives by widening perceptions on the human condition. Thus..answer question.












In The Tempest, Shakespeare highlights the power of discoveries to unveil the faults of individuals, an instrumental step in engendering transformation and repentance. From the outset, the tempest which shipwrecks the characters on the island, acts as a physical manifestation and symbol of Prospero’s flawed desire for vengeance. The plight of the characters facilitates Miranda’s self-discovery of compassion, her emotive language “If by your art... I have suffered/ With those that I saw suffer!”” acts as a foil to Prospero’s enthusiastic exclamation ”Why, that’s my spirit” in apprehending the destruction he has contrived. As such, Prospero relegates to the immorality of Alonso, Sebastian and Antonio, conspirators of his usurpation who ultimately realise their moral shortcomings through the dramatic device of a vanishing banquet followed by the pagan image of a harpy, symbol for truth and justice. This confronting discovery of the supernatural coupled with Ariel’s accusatory tone “you are three men of sin”, prompts Sebastian and Antonio to run off in a mad fit. In contrast, Alonso acknowledges his guilt and repents, using the metaphor of a church orchestra to describe the purifying experience, where “the thunder, that deep and dreadful organ pipe, pronounced the name of Prosper”. Ironically, it is through Ariel’s pathos for the characters “if you beheld them now, your affections would become tender” that Prospero transforms and recognises that “the rarer action is in virtue than in vengeance” with the antithesis of “virtue” and “vengeance” highlighting his discovery of forgiveness. Thus, The Tempest reveals the potential of discoveries to uncover flaws within individuals, laying the foundations for change and transformation.





In The Tell Tale Heart, Poe conversely depicts the limitations of discoveries to induce positive transformations through the protagonist’s lack of penitence despite apprehending his moral vices. Throughout the narrative, Poe employs the recurring motif of a heartbeat as a physical manifestation of the narrator’s internalised guilt. The succession of short sentences in “They hear!-they suspected!-they KNEW!” emulates the rhythm of the heartbeat, allowing the audience to empathise with the narrator’s troubled conscience. Thus, Poe effectively heightens dramatic tension, adding emphasis to the epiphany “Suddenly, I knew that sound was not in my ears, it was not just inside my head!”, as the narrator, like Alonso is confronted by the immorality of actions. This self discovery, catalysed by symbols of moral righteousness, the “three officers of the police”, parallels Alonso’s realisation of his immorality in confronting the harpy. However, antonymous to the peaceful denouement in The Tempest, epitomised by the dramatic device of Prospero’s unifying circle, representing the complete cycle of change and repentance, The Tell Tale heart is devoid of meaningful transformations. The narrator’s lack of contrition despite realising his moral corruption is underscored by the exclamatory repetitions in “Why does his heart not stop beating?! Why does it not stop?!”, wherein the anaphora “why” contradicts the fulfilling nature of discoveries, as experienced by Alonso. As such,The Tell Tale heart serves a counterpoint to the positive transformative ramifications of discoveries.





In The Tempest , the act of discovering new ideals entails a challenge to pre-existing beliefs, leading to the creation of renewed perspectives of the world. Shakespeare employs the sea voyage in act 1 as a historical allusion to the age of discovery during which colonialism was spurred on by expansionist ideologies. Caliban’s harsh emotive language and parenthesis “I loved thee, and showed thee all the qualities of the isle....cursed bet that I did so” establishes the notion of exploitation reflected in the relationship between Caliban and Prospero, master and slave. Within the imperialist construct, Caliban’s subjugation to the civilised Prospero is justified, where Caliban’s characterisation as a “freckled welp, hag-born...not honoured with human shape” echoes supercilious European attitudes towards natives of the “New World”.  Shakespeare utilises a parody of the colonialist role in the parallel plot of Trinculo and Stephano as they encounter Caliban to reiterate the link between physical discoveries and exploitation despite simultaneously satirising colonial theory. However, through this unexpected encounter, the audience discovers Caliban’s nobility, illustrated by the use of iambic pentameter and eloquent language “be not afeared. The isle is full of noises, sounds and sweet airs”, which contrasts his previous characterisation. The discovery of Caliban’s true nature forces the audience to question both the morality of colonialism that enslaves such noble creatures and, thus preconceived assumptions of European dominance. Therefore, the play, an allegory for 17th century European colonisation, intrinsically serves as a catalyst for the audiences’ self-discovery by challenging entrenched ideologies of the “Old world” and facilitating new perspectives on the “New World”.




Whereas discoveries in The Tempest challenge widely held assumptions of the world, discoveries in The Tell Tale Heart reshape perspectives on humanity. The psychological horror story, written in the style of dramatic monologue and first person narration invites the audience to vicariously experience the narrator’s journey of self-discovery. Initially, the narrator’s attempt to assert his sanity through repetitive rhetorical questions “Why do you say that I am mad?...Is it not clear that I am not mad?” produces an antithetical response, with the repetition of the double negative “not” demonstrating the narrator’s lack of mental coherence. The metaphoric “vulture eye”, symbolic of man’s desire for truth, expedites the constructed discovery of the narrator’s insanity, whereas in The Tempest, the audience unexpectedly discovers Caliban’s nobility. This intellectual realisation challenges the protagonist’s self-perspective and catalyses the murder of the “old man”. As such, Poe utilises the intentionally anonymous narrator as a representative of humanity, revealing the innate evil within mankind, a discovery mirrored by the contrast between Caliban and Antonio in The Tempest. Whilst civilised Antonia appears superior to Caliban, he in fact the morally corrupt “savage”, evinced by the hyperbole “twenty consciences that stand ‘twist me and Milan”. Thus, the audience’s changed perception of humanity, as facilitated by The Tell Tale Heart, parallels reshaped attitudes towards European superiority and colonialism in The Tempest, reiterating the power of discovers to challenge preconceived beliefs and create fresh perspectives. 






Inevitably, the process of discovery entails challenges to pre-existing values, namely of humanity in The Tell Tale Heart and of the colonial theory in The Tempest. Although, discoveries may develop opportunities for transformation and change, the dichotomous ramifications demonstrated in The Tempest and The Tell Tale Heart attest towards the individualised and unique nature of discoveries. Nonetheless both texts highlight....Answer question
1st in HSC Eco 2016

literally lauren

  • Administrator
  • Part of the furniture
  • *****
  • Posts: 1699
  • Resident English/Lit Nerd
Re: English Advanced Essay Marking (Modules Only)
« Reply #64 on: January 27, 2016, 10:08:40 pm »
Experiences of discoveries either challenge or affirm deeply held values, resulting in transformations and broadened perceptions. Within Shakespeare’s tragicomedy The Tempest, lies an exploration into the transformative power of discoveries supplemented by an examination into bit of repetition in the sentence structure here. Try to vary your expression the relationship between one’s self and one’s world, enabling Shakespeare to renew perspectives on colonialism this is a bit too general? What perspectives are being renewed? Are you saying he was renewing ideals that had been lost to Jacobean England? Or are you saying that the perspectives he depicted renewed those of his time? I know this is a generalised essay, but this probably wouldn't cut it in the exam as it's a little too insubstantial. However, the multi-faceted nature of discoveries espouses this word usually only works with an active agent as the subject of the sentence. Think of it like the verb 'adopt' as in 'to adopt the view that...' You can't say 'discoveries adopt the view that ramifications are placed on individuals' because discoveries can't 'adopt' anything - nor can they 'espouse.' What you seem to be getting at here is that the nature of discoveries reveals/shows that... etc. It's worth looking up some synonyms for these kinds of verbs though, as they come up a lot the differing ramifications imposed upon individuals, as Edgar Allen Poe’s psychological short story, The Tell Tale Heart, conversely depicts the limitations of discoveries in evoking change.  Nonetheless, Poe similarly creates fresh perspectives by widening perceptions on the human condition. Thus..answer question. see end comments re: memorised material

In The Tempest, Shakespeare highlights the power of discoveries to unveil the faults of individuals - an instrumental step in engendering transformation and repentance. From the outset, the tempest which shipwrecks the characters on the island, acts as a physical manifestation and symbol of Prospero’s flawed desire for vengeance. This is a good point, and you've linked a major piece of evidence to some analysis quite nicely, but there's not quite enough of a connection between this and your topic sentence; namely, what does this have to do with discovery? The plight of the characters Which characters? What plight? This doesn't link to the previous point about Prospero at all, it seems. There is a connection here, but you can't get marks for it unless you make it explicit facilitates Miranda’s self-discovery of compassion, her emotive language “If by your art... I have suffered/ With those that I saw suffer!”” acts as a foil to Prospero’s enthusiastic exclamation ”Why, that’s my spirit” in apprehending the destruction he has contrived. As such, Prospero relegates to the immorality of Alonso, Sebastian and Antonio, conspirators of his usurpation who ultimately realise their moral shortcomings through the dramatic device of a vanishing banquet followed by the pagan image of a harpy, symbol for truth and justice. There's a bit too much going on in this sentence, and it feels like you've moved away from the focus of your paragraph quite rapidly. This confronting discovery of the supernatural coupled with Ariel’s accusatory tone “you are three men of sin”, prompts Sebastian and Antonio to run off in a mad fit. In contrast, Alonso acknowledges his guilt and repents, using the metaphor of a church orchestra to describe the purifying experience, where “the thunder, that deep and dreadful organ pipe, pronounced the name of Prosper”. Good analysis! And awesome quote integration to back it up :) Ironically, it is through Ariel’s pathos for the characters “if you beheld them now, your affections would become tender” that Prospero transforms and recognises that “the rarer action is in virtue than in vengeance” with the antithesis of “virtue” and “vengeance” highlighting his discovery of forgiveness. I think you could spell this out a little more; how is this antithesis created? ie. how do you know it's an antithesis? Are you wanting to analyse the meaning of the words individually, or the fact that there's alliteration here? And how does this antithesis link with the idea of discovery? Don't just tell me 'this evidence highlights this idea' - explain why this is the case! Thus, The Tempest reveals the potential of for discoveries to uncover flaws within individuals, was this your sub-argument here? Your concluding sentence is really nice, but it feels a bit divorced from what's going on in the actual paragraph. You need to a) make sure your evidence links from point to point - there are moments where you do this well, but the beginning of the para. jumps around a bit too much, and b) ensure that you can forge a link between this collective evidence and the point you're making. What you've got here is a very solid argument, and I can see how your evidence would support it, but if I'm your assessor, I'm not allowed to give you marks for what I assume or project onto your essay; I can only credit you with what you make apparent laying the foundations for change and transformation.

In The Tell Tale Heart, Poe conversely depicts the limitations of discoveries to induce positive transformations through the protagonist’s lack of penitence despite apprehending his moral vices. what do you mean by this exactly? Are you talking about his qualms with killing, or his guilt? I'm not too sure what 'apprehending' is doing in this context or which 'vices' you're referring to. Throughout the narrative, Poe employs the recurring motif of a heartbeat as a physical manifestation of the narrator’s internalised guilt. Good! The succession of short sentences in “They hear!-they suspected!-they KNEW!” emulates the rhythm of the heartbeat, allowing the audience to empathise perhaps 'pity' would be more appropriate here. 'Empathise' implies we look at his situation and think 'ah, that sucks, and I'd hate to be in the same situation because that's how I'd respond if it happened to me' (e.g. empathising with a friend crying because they failed a test when you know what that feels like) whereas pity is more like 'that really sucks that you got yourself into this situation - I would've acted differently and in accordance with my own morals, but I still feel bad for you' (e.g. pitying a friend who got caught cheating in a test, even though you would never even consider cheating) with the narrator’s troubled conscience. Thus, Poe effectively heightens dramatic tension, adding emphasis to the epiphany “Suddenly, I knew that sound was not in my ears, it was not just inside my head!”, as the narrator, like Alonso is confronted by the immorality of actions. Really nice link here!!! This is a great way to bridge across texts and I like how subtly you transition :) :) This self discovery, catalysed by symbols of moral righteousness, the “three officers of the police”, parallels Alonso’s realisation good evidence + language here of his immorality starting to overuse this word a bit - find some synonyms! in confronting the harpy. However, antonymous an 'antonym' is a word that's the opposite of another (e.g. 'bad' is an antonym of 'good') so you could only call two things antonymous if you were talking about words and their meaning. You can't really apply it to concepts or plot devices, so go for something like 'contrary' or 'as opposed to' here instead to the peaceful denouement in The Tempest, epitomised by the dramatic device of Prospero’s unifying circle, representing the complete cycle of change and repentance, The Tell Tale Heart is devoid of meaningful transformations This is the second piece I've read that makes this argument, and I'd have to disagree here - the whole story is like one long, slow, gradual realisation, changing the character from a cold-blooded killer to a guild-ridden shell of a man who ends up dobbing himself in, which seems like a substantial transformation to me. The narrator’s lack of contrition despite realising his moral corruption is underscored by the exclamatory repetitions in “Why does his heart not stop beating?! Why does it not stop?!”, wherein the anaphora “why” contradicts the fulfilling nature of discoveries, as experienced by Alonso as in, Tell Tale's narrator only has questions, whereas Alonso is able to find answers? If that's not it, I'm not sure what your point is here. As such,The Tell Tale Heart serves a counterpoint to the positive transformative ramifications of discoveries Okay, so are you saying TTH suggests that discoveries can lead to negative transformations, or that sometimes discoveries don't lead to any positive transformations at all? There's some great discussion in this paragraph, but your final point is a little less clear - tighten that up so you can do your paragraph justice.

It'd be great if you had some linking words for the starts of these paragraphs. It can seem superficial, but even a basic 'Furthermore' or 'Similarly' can create the illusion of flow and make it seem like your paragraphs are building on one another rather than being three or four entirely separate discussions. In The Tempest , the act of discovering new ideals entails a challenge to pre-existing beliefs, leading to the creation of renewed are they 'new' or 'renewed?' The former implies you're discovering entirely new ideals for the first time, but the latter implies you're rejuvenating your perspective by seeing something familiar in a fresh light. You've got both in this sentence, which is a little confusing perspectives of the world. Shakespeare employs the sea voyage in act 1 this discussion feels like it belongs more with the first bit of your first paragraph when talking about the actual tempest and the journey itself as a historical allusion to the age of discovery during which colonialism was spurred on by expansionist ideologies. Caliban’s harsh emotive language and parenthesis “I loved thee, and showed thee all the qualities of the isle....cursed bet that I did so” establishes the notion of exploitation reflected in the relationship between Caliban and Prospero, master and slave. Within the imperialist construct, Caliban’s subjugation to the civilised Prospero is justified, where Caliban’s characterisation as a “freckled welp, hag-born...not honoured with human shape” echoes supercilious European attitudes towards natives of the “New World”. Good. Shakespeare utilises try not to overuse this word - it's not wrong, but it's a missed opportunity to say something more than 'Shakespeare uses X' Consider: 'celebrates, condemns, condones, propounds, vilifies etc.' a parody of the colonialist role in the parallel plot of Trinculo and Stephano as they encounter Caliban to reiterate the link between physical discoveries and exploitation despite simultaneously satirising colonial theory quote? However, through this unexpected encounter, the audience discovers Caliban’s nobility, illustrated by the use of iambic pentameter and eloquent language “be not afeared. The isle is full of noises, sounds and sweet airs”, which contrasts his previous characterisation. The discovery of Caliban’s true nature forces the audience to question both the morality of colonialism that enslaves such noble creatures and, thus preconceived assumptions of European dominance. Very good, this is much more clearly spelled out - aim for this kind of clarity when explaining evidence because this is excellent. Therefore, the play, an allegory for 17th century European colonisation, intrinsically serves as a catalyst for the audiences’ self-discovery by challenging entrenched ideologies of the “Old world” and facilitating new perspectives on the “New World” V good para conclusion.

Whereas discoveries in The Tempest challenge widely held assumptions of the world, discoveries in The Tell Tale Heart reshape perspectives on humanity.The psychological horror story, written in the style of dramatic monologue and first person narration invites the audience to vicariously experience the narrator’s journey of self-discovery I thought you said before that there was no meaningful transformations in the story? Initially, the narrator’s attempt to assert his sanity through repetitive rhetorical questions “Why do you say that I am mad?...Is it not clear that I am not mad?” produces an antithetical response, with the repetition of the double negative “not” demonstrating the narrator’s lack of mental coherence good point :). The metaphoric “vulture eye”, symbolic of man’s desire for truth, expedites the constructed discovery of the narrator’s insanity yes it does, but you need to tell me how and why it does so. What function does the vulture eye play in TTH? Why is this significant? whereas in The Tempest, the audience unexpectedly discovers Caliban’s nobility. I like that you're making links across the texts, but this one comes across as a bit too brief. Either flesh it out for another sentence so you can make your point properly, or just get rid of this reference and focus more so on TTH in this para. This intellectual realisation challenges the protagonist’s self-perspective and catalyses the murder of the “old man”. As such, Poe utilises the intentionally anonymous narrator you could even make something of the use of 1st and 2nd person throughout, as in 'You think me mad' where the protagonist addresses the reader directly; consider the effect of this as a representative of humanity, revealing the innate evil within mankind, a discovery mirrored by the contrast between Caliban and Antonio in The Tempest. Whilst civilised Antonia appears superior to Caliban, he in fact the morally corrupt “savage”, evinced by the hyperbole “twenty consciences that stand ‘twist me and Milan” good. Thus, the audience’s changed perception of humanity, as facilitated by The Tell Tale Heart, parallels reshaped attitudes towards European superiority and colonialism in The Tempest, reiterating the power of discovery to challenge preconceived beliefs and create fresh perspectives. Some excellent comparison here, but I think the point of this paragraph undercuts the one you made in your 2nd a bit. There's some inconsistency with your interpretation of TTH that needs to be ironed out - everything on The Tempest is fine here though.

Inevitably, the process of discovery entails challenges to preexisting values you've used this expression at the start of your 3rd para already; vary this, namely of humanity in The Tell Tale Heart and of the colonial theory in The Tempest. Although, discoveries may develop opportunities for transformation and change, the dichotomous ramifications demonstrated in The Tempest and The Tell Tale Heart attest towards the individualised and unique nature of discoveries. Nonetheless both texts highlight....Answer question see end comments

You've got some really high-quality textual analysis here, and your integration and discussion of quotes and other evidence was pretty spot-on.

First point of improvement: the ideas about discovery that you're extracting from The Tell Tale Heart seem a little bit contradictory. Put simply, it boils down to whether or not you think the protagonist does discover something over the course of the story. I'd probably argue that he does, even if he's too insane to realise it, but I could understand how someone might argue the opposite. Either way, you have to be consistent!

It's probably worth sorting this out prior to writing more essays though, so maybe go back to the text, reevaluate your stance, and just list a bunch of basic questions and answers like:
  • Does the protagonist discover anything?
  • If so, what? If not, why not?
  • What facilitates this discovery, or what hinders it?
  • Is this discovery deliberate?
  • Does this discovery change anything?
  • What effect does this discovery have on the character/plot?
  • What kind of discoveries do the readers make?
  • How do our discoveries differ from the protagonist's?
...and so on

If you find this helpful, you could do the same for The Tempest and see how many questions you can list, but most of your analysis seems fine here, so it's up to you.

This should assist you in getting a really clear-cut grasp on what the text has to say, and then you can jazz it up in your essays to present in a more 'English-y' way.

Now, with regards to pre-learned material, I totally get the impulse to memorise, but the truly high scoring students are the ones who do it in a smart way by having a 'swiss cheese' kind of essay. This means that rather than having an introduction where the first four or five lines are just the same generic statements about the human condition or the multifaceted nature of discovery (since a lot of assessors will see right through that) you'll instead have gaps every second sentence or so for you to fill in a viable connection to the prompt.

So rather than waiting until the end of your introduction to 'answer the question,' it'll be uber-impressive if you're doing that right from the start, instantly separating yourself from the pack of students who are just churning out the same old thing. It's important to ensure you're writing refined, polished sentences, but it's even more important to be addressing the prompt, so if you can make that your goal right from the start, the assessors will notice.

It's all about finding the right blend of 'stuff you've written before that you know works well and sounds nice' and trusting your ability to improvise and deal with unseen material effectively.

Also, there were sections of your piece that demonstrated a great process of dissecting evidence, explaining its relevance, and gradually building an argument. But there were others that skipped some of those steps, and it's very tough for your teachers to give you credit if you're leaping all over the place without 'showing your workings,' so to speak. Try to minimise the amount of 'X represents Y' sentences and instead opt for more in depth explanations of how and why X represents Y, if that makes sense.

Expression and sophstication really don't seem to be an issue here; there were a few odd word choice errors but nothing too significant (~mostly just me being nit-picky if I'm honest). But you've flagged this as an area of concern, so perhaps try and be more specific with yourself - what kind of expression issues do you find weigh you down?

If in doubt, go for the catch-all question for self-evaluation in English:
   Do I not know what to write, or do I not know how to write it?


So to sum up:

1) Iron out that Tell Tale interpretation so you can write about it confidently and without fear of contradiction. Question the text if you need to, as this is a good way to force yourself to come at it from a wider variety of angles and, in doing so, will potentially unlock some new essay fodder for you to discuss.

2) Make sure your essays prioritise the prompts, and practice adapting your knowledge of the texts to specific questions. Create some 'swiss-cheese' sentences that let you slot ideas into place, but don't fall into the 'chuck of rote-learned stuff' trap if you can avoid it

3) Focus on building points within paragraphs, both in terms of logic and linking sentences. I've marked the places in your piece where you were doing this well, so take note of those and replicate the same kind of approach in future

4) See if you can self-mark in order to find instances of what you believe to be issues with expression or sophistication. You could even keep a running tally when you write your next piece of all the words and sentences you find tough to write, or consider kind of clunky. Even just asterisking the margins next to each one would suffice. Then you can come back to them and (either yourself, or with the help of teachers/AN) dissect the problematic elements and improve them. There are other, more general exercises to improve expression like wider reading, but the one outlined above is probably going to be the most efficient way to tackle things.

Let us know if you have any questions, and great work overall! :)

Spencerr

  • Forum Regular
  • **
  • Posts: 98
  • Things will turn out better than expected.
Re: English Advanced Essay Marking (Modules Only)
« Reply #65 on: January 27, 2016, 10:55:22 pm »
Thanks you so much for the feedback  :) :) :)

Just a few points of clarification:

"Try to minimise the amount of 'X represents Y' sentences and instead opt for more in depth explanations of how and why X represents Y, if that makes sense."

Could you elaborate a bit more on this part? The style i adopt when I write an essay usually means I build up an argument over the course of a paragraph instead of having individual discrete sentences. Could you show me an example on how I could improve with regards to your feedback?

THANKS :) :)
1st in HSC Eco 2016

literally lauren

  • Administrator
  • Part of the furniture
  • *****
  • Posts: 1699
  • Resident English/Lit Nerd
Re: English Advanced Essay Marking (Modules Only)
« Reply #66 on: January 28, 2016, 07:52:17 am »
"Try to minimise the amount of 'X represents Y' sentences and instead opt for more in depth explanations of how and why X represents Y, if that makes sense."

Could you elaborate a bit more on this part? The style i adopt when I write an essay usually means I build up an argument over the course of a paragraph instead of having individual discrete sentences. Could you show me an example on how I could improve with regards to your feedback?
The overall paragraph argumentation is important, but you also need to make sure the individual pieces of your analysis fit together and aren't assuming too much of your assessor.

Take the following sentence: Poe foreshadows the protagonist's fate when he has him acknowledge that the old man "sitting up in the bed listening; just as [the protagonist had] done, night after night, hearkening to the death watches in the wall."

That's quite a long quite, but I'm providing the whole thing for context here.
My reasoning is that Poe deliberately mirrors the protagonist closing in upon his victim with the idea of sealed fates and encroaching darkness with the notion of the protagonist's own sense of impending doom, which reaches its zenith at the story's end. In that quote, the character states that the old man would lie awake listening to death "just as I have done," so this parallel between the soon-to-be-victim and the protagonist serves as a kind of foreshadowing for the final moments of the text. When the protagonist kills the old man, he seals both of their fates, in a way.

Now that you know my reasoning, you can see why that sentence makes sense. The trouble is that I haven't actually put any of that reasoning into my analytical sentence; I've just stated that Poe foreshadows the character's demise by having the protagonist acknowledge that both he and the old man lay awake fearing death. Or, in other words, Poe does Y, as represented by X.

But how/why does X represent Y? How/why does that quote, and the fact that there is this mirroring between the two characters, serve as a foreshadowing of the protagonist's fate?

The answer to those questions would have to be embedded in my analysis somewhere if I wanted write as safe a piece as possible. Not every time, mind you - you're allowed to assume some things, and not every point will warrant this kind of spelling out, but you should be able to pick out the important parts of your reasoning and demonstrate them in your analysis, rather than leave it all out and assume your assessor can follow your train of thought.

At this point, it'd be better to overexplain than underexplain, so perhaps pick out a few parts of your analysis, and just focus on spelling out the link between your evidence and the overall idea or argument you're building up to. If it turns out you've gone too far, then you can just cut a few bits out, and you're done, but that process will give you some idea of how much analysis is needed in certain circumstances.

Put simply, if I can still ask: 'how do you know?' at the end of a sentence in your body paragraph, then you need to go back and work on it.
& if I can ask 'so what?' at the enc of the paragraph overall, then you need a stronger, more relevant concluding sentence. The former is more important for you now, but this one can also be a good test to conduct yourself :)

Hope that clears things up!

ssarahj

  • ATAR Notes NSW MVP - 2016
  • Forum Obsessive
  • ***
  • Posts: 288
  • seek the full depth and height of life.
Re: English Advanced Essay Marking (Modules Only)
« Reply #67 on: January 28, 2016, 06:24:03 pm »
Hey it would be really awesome if you guys could have a look at one of my Discovery essays that I quickly threw together the other day.... :)
The question was, "Analyse how discoveries have a meaningful impact on a person’s sense of self".


An individual’s sense of self and identity is directly impacted by discoveries in their lives, which become meaningful due to their personal nature. This act of self-discovery can help an individual develop their relationships and perception of their purpose in the world. The play Away written by Michael Gow and the film Mao’s Last Dancer directed by Bruce Beresford both consistently demonstrate this idea. These impacts on sense of self can be a result of an individual overcoming grief, altering world views, developing a connection with nature and also finding freedom. Therefore discoveries have a very meaningful impact on a person’s perception of themselves.

Self-discovery can assist an individual in overcoming hardships and loss, fostering a deeper inner understanding. People who have dealt with grief at some point in their lives can have a stronger sense of self and resilience. In Away, the character Coral is immobilised by the death of her son in the Vietnam War to the point where she struggles to function in everyday society. She describes her pain as being, “everywhere, isn’t it? In the air we breathe”, in which Gow uses a rhetorical question and a metaphor to demonstrate the all-consuming and hopeless nature of her grief. As the play progresses towards its conclusion, Coral begins to heal as she discovers her ability to continue everyday life without her son. The Shakespearean device of a play-within-a-play, ‘The Stranger on the Shore’ is symbolic of Coral’s return from the, “silent bottom of the deep”, to, “[her] own world and [her] own people”. She demonstrates this meaningful transition by stating, “[in her own voice] I’m walking, I’m walking, I’m walking”, representing her new identity and rediscovery of the value of life. Thus it can be seen that discoveries have direct impact on an individual’s sense of self in overcoming grief and loss.

The discovery of contrasting cultures and worldviews allow for an individual to examine their identity and place in society. Often it can come as a shock for people as they experience a country or culture’s ideologies for the first time. This is very prevalent in Mao’s Last Dancer as Li is exposed to the predominantly Capitalist America which is in conflict with the Communist China he grew up in. As a child Li was effectively brainwashed into thinking that Americans, “live in darkness with hardly any daylight”. This hyperbolic statement is juxtaposed with a shot that follows of Li in an American nightclub with a close up of the bright lights surrounding him. Beresford clearly demonstrates that Li’s perceptions of the world outside of China are being challenged through his discovery of American culture. As Li spends more time in America he begins to show doubts about his home country and the Communist ideals. He states, “China [isn’t] so easy. [They] tell you what to do, where to go, what [you] can say”, which uses short phrases of dialogue to emphasise the basic aspects of his freedom that he didn’t have in China. This realisation has a major impact on Li’s worldview and ultimately leads to his defection from China. Thus it can be seen that a person’s examination of their sense of self can be as a result of the discovery of different cultures and worldview.

Nature can assist and act as the catalyst for discoveries that impact an individual’s sense of self. The powerful ability of nature as a physical and metaphorical presence allows for many meaningful discoveries. In Away, Gwen is transformed from being obsessed with structure and material possessions to having appreciation for family and the value of life. The act of going away from home on a holiday provides her with the opportunity for self-discovery without the pressure of domestic spaces. This is compounded by Gow’s use of the Shakespearean device of a storm to put Gwen in a position of vulnerability with the destruction of her possessions, such as her, “new caravan. With everything in it you could want”. The beach is also used as a symbolic place of healing as this is where Gwen discovers the importance of family and her relationships after presumably learning of Tom’s illness from Vic. Her transformation is epitomised through her attempt to reconnect with her husband Jim as she says, “Come on, down to the water. The water’s so warm”. The symbol of water as rejuvenating and calming helps the audience to understand Gwen’s focus on reconciliation.. Therefore it is clear that the influence of nature can lead to discoveries which have meaningful impacts on individuals.

A person’s discovery of freedom in their life can promote a stronger perception of themselves. This is even more powerful after being in an repressive situation. In Away, Tom is able to come to terms with his imminent death despite his parents refusal to acknowledge the terminal nature of his illness. This allows Tom to discover the freedom of accepting his destiny. This is epitomised in his final line of the play, “Unburden’d crawl towards death”. The intertextuality of this passage from Shakespeare’s King Lear adds depth and context to Tom’s situation and symbolises the conclusion of his struggle with his mortality. Similarly in Mao’s Last Dancer, Li discovers his identity and true capability through the freedom he finds dancing in America. In Li’s first ballet production in Texas he performs a complicated solo jump which is shown in slow motion as a mid-shot of his chest with arms outstretched and a crescendo in the music. This represents him breaking free of his autocratic Chinese Communist past and embracing his future in America. Therefore it can be seen that an individual’s discovery of freedom in their lives can enhance their sense of self.

The discoveries an individual makes throughout their life have meaningful impacts on their sense of self and the type of person they become. Discovery as a result of grief, differing cultures, the influence of nature and freedom from repression provides an individual with opportunity for self-reflection, leading to a better understanding of themselves. Thus it can be concluded that the personal nature of discoveries in an individual’s life provide meaningful impacts on their sense of self. 
HSC 2016: SOR 2, Adv. English, Ext. 1 English, Chemistry, 2U Maths, Hospitality
6th in NSW for SOR 2

ATAR: 97.00

2017: B Nutrition & Dietetics @ University of Newcastle

MemeKing

  • Adventurer
  • *
  • Posts: 6
  • School: James Ruse
  • School Grad Year: 2016
Re: English Advanced Essay Marking (Modules Only)
« Reply #68 on: January 29, 2016, 04:59:34 pm »
Hey!!  Could you please look at my essay?  I didn't too well in the last assessment and would appreciate feedback :)

Rediscovery -Rediscovering Hurley’s Artworks in order gain a more appreciative and heightened meaning
Nasht’s cinema verite of Hurley traces the mutability of his artworks to persuade an audience to reassess and appreciate the historical truth of his manipulations.  Nasht configures Hurley in ‘Frank Hurley’ to serve as a consistent reminder to the audience of his transience through the initial montage of Hurley’s images in accompany with dramatic noble music.  This attempts to immortalize the otherwise ephemeral significance of Hurley’s artworks in order to convey to an audience of Hurley’s “remarkable photography” and to view Hurley in a different perspective.  In an attempt to prevent the decay of Hurley’s legacy, Nasht attempts of justify Hurley’s otherwise immoral representation of reality in a purist society by employing a series of quadtiptyches.  This is cultivated in order to contest the inevitable subjectivity of the historical truth behind Hurley’s images and presents an arbitrary audience the rationale behind Hurley’s manipulations.  Furthermore, Nasht’s embellishment of Hurley coincides with the postmodern pluralist disposition of “a world searching for heroes” and furnishes a now accepting society the ability to question historical truth.  This is displayed further through Nasht’s portrayal of the cyclical nature of the auction house in accompany with “The Polar Sale” that ultimately commemorates the appreciation and value of Hurley’s artwork once a societies’ rationale transcends pre-existing paradigms.  Ultimately in ‘Frank Hurley’, Nasht conveys to an audience that through discovering a more justified approach to questioning historical truth can a more appreciative stance on historical truth be gained.

Self-Discovery – Discovering Hurley’s inability to sustain his ‘showman’ facade and thus attempts to perpetuate the creation of himself as a myth
Upon reassessing one’s inability to sustain one’s desired facade, man attempts to perpetuate their own fallacy in an attempt to succour their desired self-image.  Hurley’s incapability to support the perpetuation of his legacy is highlighted through Nasht’s depiction of Hurley’s life as a personified “story” embellishing “many stories” but his “own”.  This suggests that Hurley tried consistently to portray himself as his own myth, however, could not sustain his depiction and ultimately resorted to associating himself with other stories.  However, in “Frank Hurley”, Nasht engages in demonstrating to an audience of Hurley’s self-discovery that ultimately provokes him to undergo inherent transformation that eventually leads to the creation of a myth that perpetuates Hurley’s decide self-image. Nasht’s attempts to symbolise Hurley’s desired legacy is demonstrated through constant mythological symbolisms to “the hero’s journey”.  Nasht engages in this narration in order to portray Hurley’s inherent desire to perpetuate himself to a purist audience of his own mythological fallacy.  His inability to perpetuate his desired facade renders him “exhausted by the struggle”, however, Hurley’s ambition towards self-fulfilment is highlighted through Nasht’s placement of the final voiceover of Hurley’s diaries stating, “If I could live my life again, I’d do it all exactly the same”.  This final voiceover in conjunction with images of Hurley with a camera is ultimately placed by Nasht in order to immortalize Hurley’s fallacy as a “grand illusionist”.  Ultimately, Nasht demonstrates Hurley’s ability to reassess him and undergo a process that effectively allows Hurley to sustain and perpetuate his desired self-image.

lauren7366

  • Fresh Poster
  • *
  • Posts: 1
  • School: Gosford high school
  • School Grad Year: 2016
Re: English Advanced Essay Marking (Modules Only)
« Reply #69 on: January 29, 2016, 06:34:02 pm »
Heres my essay on the tempest which is in process. just wanting to know if I'm on the right track and if the sentences make sense and all. it isn't a finished product and i haven't included much about my related text yet - the count of monte cristo film

Stories of our past help us discover who we are in the present.
To what extend does this confirm your understanding that discoveries can be reassessed over time?

Discoveries can be a process of reassessment, which are catalyzed due to change of circumstance and reflection of ones past mistakes. As a result of these confronting and provocative discoveries, enlightenment is reached in the reformation of our morals, encouraging humanist values such as forgiveness and love. William Shakespeare’s pastoral comedy, “The Tempest” written in 1610, and the film, “The count of Monte Cristo,” directed by Kevin Reynolds, both exemplify the corrupt desire for vengeance that allows for a renewed perception of world view once it is realized to be deceitful. This realization comes through reflection of the past in order to progress in future endeavors.

The Tempest follows the reformation of an individuals absolute control and power, into benevolent traits such as internal judgment and empathetic characteristics that would thrive within utopian governance. This is achieved by self acceptance of past faults providing a renewed perception of compassion and forgiveness; notions that are the fundamentals of humanism. When remarking to Miranda the “foul play”, that they were “heaved thence,” Prospero highlights the corrupt society in which he ruled, where he himself “neglect[ed] worldly ends, all dedicated to the closeness and the bettering of [his] mind,” but also the world in which he was betrayed by “ a brother […] so perfidious.” Shakespeare utilizes this as a microcosm for the power lust that humanity exemplifies in positions of authority; knowledge prized over our collective morality and familiar bonds. The 1600’s imperialistic context of control and influence over the majority, is prevalent within Prospero’s need to manipulate native Caliban and Ariel’s servitude for personalized advantage. This is showcased using the disruption of natural order, the tempest itself symbolizing the brazen attitude that Prospero subjects’ others to; as a result of his ultimate thirst for revenge that dominates all cognitive deliberation. Audiences grapple with Prospero’s shortcomings to understand that life manipulates the circumstances that we oversee, however it is how we confront these circumstances that determines our future success.

Individuals in powerful positions often have to experience unprompted failure in order to reassess corrupt core values and furthermore access personalized freedom and amend flawed leadership qualities. It is through Prospero’s relationship with spirit Ariel, that access to worldly insight is obtained; from self absorption into acceptance of the altruistic experiences in life, taking from this a renewed optimism for the future. Prospero’s individual values of hatred, are challenged by Ariel’s perspective of tenderness and wisdom in the face of choice, “that if you now beheld them, your affections would become tender.” This is then furthered by the acknowledgment of the humanistic characteristics that Prospero should adhere to by reason of his biological makeup, “mine would sir, if I were human.” From this, audiences are reminded of the humanity we are answerable to exhibit in order to thrive as a society, however Prospero also learns that his corruption is what erodes humanisms core belief; if we cannot have faith in the people, a strong fundament of politics cannot be developed. We also see this realised in alliteration, “rarer act is in virtue than in vengeance”, symbolising the discovery of human connection and that kindness prevails over the corrupt quest of power.

Prospero’s epiphany similarly coincides with Edmond Dante’s experience, his integral belief in vengeance challenged by Abbe Farria (priest) and the revelation of a biological son, leading to a rewarding sense of love and peace from one’s inner demons. Chief prosecutor Ville fort; one who originally deceives Edmond into imprisonment, foreshadows the enlightenment that Edmond is to experience at the end of the film, “perhaps some good will come out of this treasonous affair.” Audiences understand this enlightenment to be that he discovers the intrinsic value of knowledge to be a tool, providing the wisdom to forgive even in the face of ultimate choice. The unpredictable discovery of his biological son, through exclamatory language accompanied by directed close up, “Albert, you are the son of Edmond Dantes. The man you know as the Count of Monte Cristo,” allows Edmond to reassess the importance of a personal vendetta when presented with opportunity to rebuild love ; a founding characteristic that is vital in order to further one’s humanism.


thanks

foodmood16

  • Adventurer
  • *
  • Posts: 10
  • School: Narara Valley High School
  • School Grad Year: 2016
Re: English Advanced Essay Marking (Modules Only)
« Reply #70 on: January 30, 2016, 10:38:56 am »
This is my discovery essay on Go Back To Where You Came From and a related. I haven't gone well in the other assessments so far, so I want to know if I am on the right track.  :)

The question is 'Discoveries can be transformative and/or far-reaching for an individual'

The emotional, intellectual and physical discoveries can act as the foundations for an individual and can have a transformative effect on their awareness of human experiences and the wider world. The discoveries can be carefully planned, so much that the composer can place an individual in unauthentic situations, even without their knowledge. In the 2011 documentary series ‘Go Back To Where You Came From’, the participants are taken on the migrant journey in reverse and the experiences of Adam Hartup and Darren Hassan show many emotional ramifications that are impacted by their awareness of the influence of the media on this political issue. The producers have seemingly composed many of the experiences to exaggerate the extent of their discoveries, making the responders question the far-reaching impact of the emotional responses of the participants. Similarly, in the 1963 poem ‘Mid-Term Break’ by Seamus Heaney explores the transformative discoveries of his own childhood, having lost an infant in traumatic accident. He focuses on how this impacted the people around the protagonist, as well as the reader. He uses carefully planned literary techniques in order to provoke an emotional response from the reader. Both texts explore how discoveries can be influenced by the composer by making the audience and protagonist aware or unaware of these influences.

The foundations of transformative discoveries can be affected by the authorial presence and careful planning of a reality TV series, and ultimately the participants are unaware of this presence. During Episode 2 of ‘Go Back To Where You Came From’, the participants take part in a police raid on a construction site. On the ride to the site, Adam comments, “if it’s the Chins I’ll lose my shit… I won’t be able to handle it.” The non-diegetic music builds into a suspenseful tone and the camera shows Adam in a close-up shot. This shows Adam’s unawareness of how his reaction to this confronting event can be manipulated by the producers. The emphatic language shows the growing attachment and understanding Adam has developed through the emotional discoveries. Throughout the raid, Adam doesn’t say much, but his body language expresses his confusion and confrontation, unaware of how to act in retaliation to the site of arrested refugees. As new discoveries are made, it becomes clear to the participants and audience how an authorial presence can shape material into their preferred ideal.

For the reader, transformative discoveries and their foundations are affected by the author of a text through their careful use of literary techniques, with allows the reader to emotionally respond to the text. In the poem, ‘Mid-Term Break’, many techniques are used to convey the story of the death of an infant, although the reader doesn’t discover this until the end of the journey. The title itself “Mid-Term Break”” leads the responder to think the poem is about a happy event of the end of the school year, making them unaware of the poignant and traumatic story that lay ahead. It is the end of the first stanza that the reader suspects that something unusual is about to occur, though still oblivious to the traumatic event. “At two o’clock our neighbours drove me home.” This unusual event of the neighbours taking the protagonist home allows the reader to prepare themselves for the death of the four-year-old boy, building the morbid mood. They are still unaware of the emotional consequences of the death. Far-reaching discoveries can be impacted by the author’s manipulation of words to lead to greater the impact of the responder’s emotional response.

As new understandings and perceptions develop from discoveries, it becomes clear how constructed the media is in order to portray a particular side of a story. The producers of ‘Go Back To Where You Came From’ have aimed to emphasise their pro-humanitarian motivation to provoke an empathetic response by the audience. At the end of Episode 1, the participants are placed on refugee boat, and are forced to travel into the open sea without knowledge of their destination. After experiencing this unauthentic situation, the live camera captures Darren, the most opposed to boat people, express his concerns that the media is “…emotionally involving us without our consent.” The collective terms used in his language shows that he is talking not only about himself, but the Australian public. Darren’s body language and tone also shows his agitation by the experience, and shows his awareness of how the program has constructed these ‘transformative’ experiences in order to provoke an emotional response by the participants. He also explains that the media wants “you should feel bad, you should feel empathy.” Darren is placed in a mid-shot, and uses high modality language to highlight his feeling of manipulation by the media. The awareness of constructed discoveries that an individual may undertake can therefore influence their shift in perceptions, or even affirm their previous understandings.

The new understandings and renewed perceptions that derive from the foundations of discoveries are impacted by the author’s choice of form and stylistic elements of their writing. In the poem ‘Mid-Term Break’, Heaney explores the parent’s reactions to the death of the infant, before the protagonist’s is discovered. In stanza 2, the father’s reaction is distinguished. “… I met my father crying…always taken funerals in his stride…” In the 1950’s, the thought of a man crying was against the society’s stereotypical man, and the use of this emphasises the lasting impact of the death of a loved one, especially an infant.  The mother’s reaction to this terrible news is expressed in stanza 5. “…coughed out angry tearless sighs.” The silence of “tearless sighs” accentuates the idea of the reverence of death. At this stage, it is still unclear to the audience of who is dead, though the characters portrayed have already discovered this. The awareness of the characters show how discoveries build to the transformative foundations of the audience and their new understandings.

In conclusion, both the SBS documentary series ‘Go Back To Where You Came From’ and the poem ‘Mid-Term Break’ explore how the audience and participants are aware or unaware of how the composer influences the discoveries they make.

literally lauren

  • Administrator
  • Part of the furniture
  • *****
  • Posts: 1699
  • Resident English/Lit Nerd
Re: English Advanced Essay Marking (Modules Only)
« Reply #71 on: January 30, 2016, 01:37:37 pm »
Comments below; let me know if you guys have any questions! :)

Feedback for ssarahj
"Analyse how discoveries have a meaningful impact on a person’s sense of self".

An individual’s sense of self and identity is should be 'are' since you're talking about both 'sense of self' and 'identity,' and the distinction between the two can be quite interesting, in fact directly impacted by discoveries in their lives, which become meaningful due to their personal nature. This act of self-discovery can help an individual develop their relationships and perception of their purpose in the world. The play Away written by Michael Gow and the film Mao’s Last Dancer directed by Bruce Beresford both consistently demonstrate this idea. These impacts what impacts, exactly? If you're talking about developing relationships and perceptions, those ideas haven't really carried into the following sentence, so you've got a discussion of some concepts, then a sentence about the texts, and then you go back to those concepts again, which can be a bit jarring to follow on sense of self can be a result of an individual overcoming grief, altering world views, developing a connection with nature and also finding freedom. Try to avoid this listing of ideas if you can. Often intros are strengthened by 'opening up' the discussion, so spending maybe a sentence on each of these in order to flesh them out would be preferable to simply getting them all out of the way in one go. Therefore discoveries have a very meaningful impact on a person’s perception of themselves.

Self-discovery can assist an individual in overcoming hardships and loss, fostering a deeper inner understanding. People who have dealt with grief at some point in their lives can have a stronger sense of self and resilience. Great! I like that you're not just jumping from a hugely broad idea about discovery into the text in the very next sentence; these transition points where you gradually 'zoom in' are awesome! In Away, the character Coral is immobilised by the death of her son in the Vietnam War to the point where she struggles to function in everyday society. She describes her pain as being, “everywhere, isn’t it? In the air we breathe”, in which Gow uses a rhetorical question and a metaphor to demonstrate the all-consuming and hopeless nature of her grief see end comments!. As the play progresses towards its conclusion, Coral begins to heal as she discovers her ability to continue everyday life without her son. The Shakespearean device of a play-within-a-play perhaps this is terminology your teacher has verified (in which case ignore this) but I wouldn't necessarily call a 'play-within-a-play' a Shakesperean technique.He wasn't the first to use it in Hamlet and intertextuality is a very common thing, so unless you want to make a more substantial link here, it might be better to alter the wording, ‘The Stranger on the Shore’ is symbolic of Coral’s return from the, “silent bottom of the deep”, to, “[her] own world and [her] own people”. She demonstrates this meaningful transition by stating, “[in her own voice] I’m walking, I’m walking, I’m walking”, representing her new identity and rediscovery of the value of life. But how does her statement of "I'm walking" represent this? The evidence you've selected and the arguments you're making here are both excellent; you just need to connect the two more clearly to ensure you're getting sufficient credit. Thus it can be seen that discoveries have direct impact on an individual’s sense of self in overcoming grief and loss. V. good para conclusion that links directly to both the discussion and the topic :)

The discovery of contrasting cultures and worldviews allows  --since it's singular 'discovery' even though you're talking about a plural contrast for an individual to examine their identity and place in society. Often it can come as a shock for people as they experience a country or culture’s ideologies for the first time. This is very prevalent in Mao’s Last Dancer as Li is exposed to the predominantly capitalist American ideology which is in conflict with the Communist China he grew up in. As a child Li was effectively brainwashed into thinking that Americans, “live in darkness with hardly any daylight”. This hyperbolic statement is juxtaposed with a shot that follows of Li in an American nightclub with a close up of the bright lights surrounding him. Beresford clearly demonstrates you're using this word quite a bit (and it's a good word to use, but overuse tends to lead to over-reliance, which can then lead to repetition) so perhaps lookk up some synonyms for this that Li’s perceptions of the world outside of China are being challenged through his discovery of American culture. As Li spends more time in America he begins to show doubts about his home country and the Communist ideals. He states, “China [isn’t] so easy. [They] tell you what to do, where to go, what [you] can say”, which uses short phrases of dialogue to emphasise the basic aspects of his freedom that he didn’t have in China. This realisation has a major impact on Li’s worldview and ultimately leads to his defection from China. Thus it can be seen that a person’s examination of their sense of self can be as a result of the discovery of different cultures and worldview. Good, though this paragraph conclusion is a little too similar to the statements you've already made. I noted in the last para. that it's good to have a concluding point that ties together the main focus of that discussion, but you don't want it to feel redundant or repetitious. Different wording can help here, but you may also want to build out to the prompt from a slightly different angle (e.g. why is it that self-reflection or exposure to other cultures lead to a change in one's worldview?)

Aim for greater fluency between paragraphs too; more on this in the end comments. Nature can assist and act as the catalyst for discoveries that impact an individual’s sense of self. The powerful ability of nature as a physical and metaphorical presence allows for many meaningful discoveries. In Away, Gwen is transformed from being obsessed with structure and material possessions to having appreciation for family and the value of life. The act of going away from home on a holiday provides her with the opportunity for self-discovery without the pressure of domestic spaces. This is compounded by Gow’s use of the Shakespearean device of a storm the 'play-within-a-play' thing was dubious, but I definitely wouldn't call the storm a 'Shakesperean' thing to put Gwen in a position of vulnerability with the destruction of her possessions, such as her, “new caravan. With everything in it you could want”. The beach is also used instead of saying 'used' which could come across as a fairly pedestrian word, try and go for a more descriptive way of describing what the author (/playwright) is trying to accomplish here. There's a great thread on improving vocabulary here for reference if you need a place to start :) as a symbolic place of healing as this is where Gwen discovers the importance of family and her relationships after presumably learning of Tom’s illness from Vic. Her transformation is epitomised through her attempt to reconnect with her husband Jim as she says, “Come on, down to the water. The water’s so warm”. The symbol of water as rejuvenating and calming helps the audience to understand Gwen’s focus on reconciliation. Therefore it is clear that the influence of nature can lead to discoveries which have meaningful impacts on individuals. I like that you're prioritising clarity in these sentences because it means I'm left in no doubt about your focus and the relevance of this discussion to the topic. Now all you have to do is vary your sentence structure, as I'm noticing each of your paragrpahs ends in a very similar way - and it's effective, but needs to not be noticeable ;) Again, consult the link above and scroll down from some alternate sentence structures if needed.

A person’s discovery of freedom in their life can promote a stronger perception of themselves. This is even more powerful after being in an repressive situation. In Away, Tom is able to come to terms with his imminent death despite his parents refusal to acknowledge the terminal nature of his illness. This allows Tom to discover the freedom of accepting his destiny. This is epitomised in his final line of the play, “Unburden’d crawl towards death”. The intertextuality of this passage from Shakespeare’s King Lear adds depth and context to Tom’s situation and symbolises the conclusion of his struggle with his mortality. Similarly in Mao’s Last Dancer, Li discovers his identity and true capability through the freedom he finds dancing in America. In Li’s first ballet production in Texas he performs a complicated solo jump which is shown in slow motion as a mid-shot of his chest with arms outstretched and a crescendo in the music. This represents him breaking free of his autocratic Chinese Communist past and embracing his future in America. Therefore it can be seen that an individual’s discovery of freedom in their lives can enhance their sense of self. By the end of this paragraph (i.e. the end of your essay) it'd be good if you were able to zoom out and make a point about the nature of discovery more broadly. You've discussed your sub-argument well, but this should be moving back out to your overall contention here. The easiest way to do that is to keep asking 'so what' until you get to that really abstract level.
i.e. 'discovering freedom enhances one's sense of self'
>> so what?
Well, freedom and self-determination are inextricably linked
>> so what?
Without the freedom to discover oneself, we would be unable to actually define and understand ourselves.
^^and that's the sentiment that goes in the end of our paragraphph.

Note that you can keep conducting this exercise over and over again to see how far it'll take you. The same can be done with other questions like 'why?' and 'how do I know?'


The discoveries an individual makes throughout their life have meaningful impacts on their sense of self and the type of person they become. Discovery as a result of grief, differing cultures, the influence of nature and freedom from repression bit list-y - avoid this provides an individual with opportunity for self-reflection, leading to a better understanding of themselves. Thus it can be concluded that the personal nature of discoveries in an individual’s life provide meaningful impacts on their sense of self. I think your conclusion is letting you down a bit here. The first and last sentences are saying almost exactly the same thing, which is a statement that's already been made numerous times throughout your piece. The clarity and connections are great, but you don't want to go too far overboard and end up having an essay that's too narrow or repetitious. Likewise the sentence that lists off the focus of your body paragraphs doesn't really unite these ideas to say something meaningful about the way they relate to one another; it's just a checklist of things that have already been explored. Try to zoom out a bit more here; the questions listed at the end of the previous para might give some frameing here.

Really good stuff here! You seem to have the underlying structure well under control, and the clarity of your expressions was pretty much flawless.

There's some improvement to be made with regards to 'spelling out' your evidence and ensuring the assessors can get from A (your quotes and textual examples) to B (your overall ideas).

Taking this excerpt from your first paragraph:
"She describes her pain as being, “everywhere, isn’t it? In the air we breathe”, in which Gow uses a rhetorical question and a metaphor to demonstrate the all-consuming and hopeless nature of her grief."
and breaking this down into its essential bits, we have:
- the quote: "[pain is] everywhere, isn't it? In the air we breathe"
- the devices: (rhetorical question & metaphor)
- the meaning: Coral's grief is all-consuming and hopeless
but how are these pieced together? You state that there is a connection (and there totally is!) but if you were to make this explicit, your assertion would be a lot stronger. So, in this case, how do you know her grief is all-consuming and hopeless? How does that quote demonstrate your point?
See this for a possible answer
If we're to focus on the metaphor part of the quote, we've got to try and get from "[pain is] in the air we breathe" to the notion of hopelessness and consumption, so what is it about this language that might lead us to this idea? When, the concept of something being "in the air we breathe" implies that it is not only all around us (=all-consuming, or at least ubiquitous) but that it also infiltrates us and becomes a part of the air. Thus, to breathe (and to live) is to experience pain, because it is so ubiquitous an unavoidable. Now we can reasonably conclude that there is an element of saddness and hopelessness in that quote because of this rationale, so your analysis might look something like:
She describes her pain as being "everywhere, isn't it? In the air we breathe" whereby Gow's metaphorical description of pain being such an omnipresent and inescapable force serves to demonstrate the all-consuming and hopeless nature of Coral's grief.
It may seem like a small change, but that one tiny step of 'showing your workings' can be very advantageous in showcasing your knowledge of the text and ability to extract ideas from it.
Also, and I believe I may have mentioned this in a previous bit of essay feedback but just in case: you want your paragraphs to build on one another. If I were to change the order around and read your 3rd para first and your second one last or w/e, it should seem weird! You don't want your essay to be an assortment of random mini-discussions that go off on their different tangents without linking together; instead, you want it to be like a pyramid with each brick aiding you in reaching that high point of your thesis statement in response to the prompt. You still have to have separate paragraphs, of course, but you should see them as a series of stepping stones to get you to an overall point, rather than being like... I don't know... flower petals that just stem out from the centre but don't combine help you get anywhere with your arguments.

Beyond that, some greater variation in sentence structure and vocabulary would also help you bolster your piece, especially when it comes to the conclusion. You always want to end with impact, as this is where the mark is decided, so to restate your points and only use sentences that have basically already been used earlier is to tell the assessor you don't have anything better to say, meaning that if they're hovering between a 7/10 and an 8/10, a mediocre conclusion might be what tips the balance to a 7 rather than an 8.

Also, and this might be a matter of some contention depending on your teacher, but I'd say it'd be great to have a paragraph (or multiple paragraphs, ideally) that talk about both of your texts instead of switching between them both. If you are to forge connections between the two, you should really talk about them together. See some of the previous corrections for advice on potential essay structures.

But you're already in the enviable position of basically having all the content right, and just needing to fine tune your presentation of it. So, the stuff you should aim to work on:

1) Have your ideas and sub-arguments build on one another. This is often best practised through essay plans as opposed to full essays, so maybe draw up some rough outlines and give yourself some idea of how things will link together to help develop your thesis statement.

2) Ensure that you're connecting your examples to your ideas as directly as you can, with the emphasis on explaining how certain evidence demonstrates your point.

3) Vary your sentence structures, and try to avoid taking your body paragraph topic sentences and just listing or rewording them in the intro, B.P. concluding sentences, and conclusion.

Very good effort overall :)

Feedback for MemeKing
Rediscovery -Rediscovering Hurley’s Artworks in order gain a more appreciative and heightened meaning.

Nasht’s cinema verite of Hurley traces the mutability of his artworks to persuade an audience to reassess and appreciate the historical truth of his manipulations word check - the other vocab in this sentence is great, but calling his artworks 'manipulations' sounds a bit odd. 'Oeuvre' is always nice if you don't mind sounding like a pretentious Frenchman :P  Nasht configures Hurley in ‘Frank Hurley’ to serve as a consistent reminder to the audience of his transience through the initial montage of Hurley’s images in accompany with accompanied by dramatic noble word check; music can't really be 'noble' music.  This attempts to it sounds a bit odd to say this when the focus of the last sentence is the montage and dramatic music. I know it seems like a minor quibble, but music can't 'attempt to' do something; you want to make the author/director your focus here using a phrase like 'This forms part of Nasht's attempts to...' or 'To this end, Nasht seeks to...' immortalize the otherwise ephemeral significance of Hurley’s artworks in order to convey to an audience a sense of Hurley’s “remarkable photography” and to view Hurley in a different perspective. This sentence is getting a bit long; don't overuse the 'and...' structure as it makes it seem like you're writing run-on sentences.  In an attempt to prevent the decay of Hurley’s legacy, Nasht attempts to justify Hurley’s otherwise immoral representation of reality in a purist society by employing a series of quadtiptyches quadtiptyches of what, exactly? You need more information about your evidence here.  This is cultivated in order to contest the inevitable subjectivity of the historical truth How do you know? In what way does the evidence you just mentioned contribute to this idea? I'm not seeing a connection, and your priority should be to make that link between examples and ideas really clear behind Hurley’s images and presents an arbitrary word check; 'arbitrary' means unimportant or random, as in, 'my school has some arbitrary rules about uniform, like the fact that ties have to be exactly 33.7cm long' :P I'm not sure what you're intending in this case audience the rationale behind Hurley’s manipulations.  Furthermore, Nasht’s embellishment of Hurley coincides with the postmodern pluralist disposition of “a world searching for heroes” and furnishes a now accepting society the ability to question historical truth. Nice sentence, but it doesn't really gel with what you're saying here. The previous sentence was about Hurley's intentions, and now you're talking about 'postmodern pluralist disposition' without clarifying what that means or how it's conveyed in his work. Basically, it's not enough to say 'the author does X which is indicative of Y' - you have to explain how you got from one point to the next, and if I were an assessor, I couldn't give you credit for this sentence because you hadn't substantiated your point with such an explanation. This is displayed further through Nasht’s portrayal of the cyclical nature of the auction house in accompany with “The Polar Sale” that ultimately commemorates the appreciation and value of Hurley’s artwork once a societies’ rationale transcends pre-existing paradigms. This is better, there's a bit more 'spelling out' here; try to do this with your evidence more often. Ultimately in ‘Frank Hurley’, Nasht conveys to an audience that through discovering a more justified approach to questioning historical truth was this what your paragraph had been building towards? Your evidence doesn't seem to be supporting this idea; what is the 'historical truth' being questioned and how is Nasht questioning it? can a more appreciative stance on historical truth be gained.

Self-Discovery – Discovering Hurley’s inability to sustain his ‘showman’ facade and thus attempts to perpetuate the creation of himself as a myth
Upon reassessing one’s inability to sustain one’s desired facade, man attempts to perpetuate their own fallacy in an attempt to succour their desired self-image.  Hurley’s incapability to support the perpetuation of his legacy is highlighted through Nasht’s depiction of Hurley’s life as a personified “story” embellishing “many stories” but his “own” I haven't seen the film but I'm assuming you either mean a) that the story embellishes other stories, but does not embellish his own, or b) that the story embellishes other stories, but is still, nevertheless, his own story (-this second one seems more likely, but I'm not sure).  This suggests that Hurley tried consistently to portray himself as his own myth, however, could not sustain his depiction and ultimately resorted to associating word check; 'obfuscating' might work here, depending on what you're trying to say himself with other stories.  However, in “Frank Hurley”, Nasht engages in demonstrating to an audience this is a fairly clunky phrase, and it feels like you're taking six words to say what could easily be summarised in one, good, punchy verb like 'celebrates' 'critiques' 'vilifies' 'extols' etc. Hurley’s self-discovery that ultimately provokes him to undergo inherent transformation that eventually leads to the creation of a myth that perpetuates Hurley’s decide self-image this sentence is getting a bit run-on-ish as well; as soon as you're using the same structure or conjunction multiple times in a row (eg. 'and... and...' or 'that... that...') it becomes a little laboured, so watch out for that. <Linking word here would be good>Nasht’s attempts to symbolise Hurley’s desired legacy is should be 'are' because we're talking about plural 'attempts' demonstrated through constant mythological symbolisms to “the hero’s journey”. expression is a bit clunky, perhaps 'the mythological symbolism of "the hero's journey".'  Nasht engages in this narration what narration? Evidence? in order to portray Hurley’s inherent desire to perpetuate himself to a purist audience of his own mythological fallacy.  His inability to perpetuate repetition his desired facade renders him “exhausted by the struggle”, however, Hurley’s ambition towards self-fulfilment is highlighted through Nasht’s placement of the final voiceover of Hurley’s diaries stating, “If I could live my life again, I’d do it all exactly the same”.  This final voiceover in conjunction with images of Hurley with a camera is ultimately placed by Nasht in order to immortalize Hurley’s fallacy as a “grand illusionist”.  Ultimately, Nasht demonstrates Hurley’s ability to reassess him who? and undergo a process that effectively allows Hurley to sustain and perpetuate his desired self-image. Good use of evidence, and nice para conclusion here :)
Very interesting discussion with some really complex ideas at play here, but you need to be careful not to let good words get in the way of great ideas, so to speak. There were times where your arguments were a little unclear of confusing, and your vocabulary is kind of exacerbating that issue rather than masking it.

In English, vocab is like an amplifier. If you're doing things well (like by spelling out your evidence and building up relevant ideas) then your vocabulary will usually magnify that good stuff and make it even better. But if there are problems with your understanding with the text (which doesn't seem to be an issue for you) or with the clarity of your ideas (- more pertinent, in this case -) then repetitious sentence structures or out-of-place words can draw attention to those concerns.

As such, I'd recommend prioritising clarity, even if it's at the expense of sophistication. It doesn't seem like you'll have any problem transforming your ideas into complex, well-worded sentences once you've sorted them out, but it's that sorting out that needs to happen first! Otherwise, it's like you're adding icing and cake decorations to a cake that hasn't been baked yet :P

I'm also a little bit confused as to the structure of your 'essay.' Maybe it's because I'm just a humble VCE-er and I don't know how your school operates, but unless this was a kind of short answer response, it seems very odd that you wouldn't be required to have a formal introduction and conclusion with three to five body paragraphs, as I know that's what the end-of-year standard tends to be. Perhaps this was a different exercise, but if you were asked to write an essay and you instead wrote two somewhat disconnected paragraphs that conveyed your ideas, that might've contributed to you're not doing too well in the last assessment.

Having said that, the structure of those paragraphs was really good; you've got the abstract unpacking of your ideas at the start, and the zoomed out summative sentences at the end, which gives things a nice, compact, and most importantly precise focus. The only thing that needs work in that regard is your linking between evidence and ideas. I've written about this quite a bit in previous essay comments, so perhaps go back and read those if you need further clarification, but in short, you need to explain how the evidence you're using supports your points. Sometimes you can get away with not doing this (i.e. not every sentence of your body paragraph has to be an incredibly detailed, lengthy discussion of why X demonstrates Y,) but it is a surefire way to impress the assessor and boost you into higher territory.

I'd also say you'd need more evidence in order to make your ideas that much stronger. Your use of quotes and metalanguage were solid, but you just need to do it more often. Both paragraphs here had one moment towards the end of really excellent analysis, and you want to be replicating that kind of quality three or four times per paragraph.

Things to work on:

1) Ensure you can explore the connections between evidence and argument as clearly as possible, and that your expression doesn't get in the way of this.

2) Keep experimenting with vocab and using new words, as this is the only way to truly acquire a better vocabulary, but look up definitions and synonyms if you're unsure, as you have the opportunity to use such resources now even though you won't in assessment tasks. I know I've isolated a couple of word checks, but honestly, I'd rather you made a hundred mistakes now and learned from each of them than made none at all and never developed your arsenal, so keep that up because your vocabulary is clearly well-developed and will be a huge strength this year if you can iron out the little things.

3) Make sure you have a substantial basis of evidence for making your claims. This goes both for the small level sentence-by-sentence assertions in that if you don't back up 90% of what you say, it's very hard to attain marks, but also on the broader paragraph-by-paragraph level in that you need breadth as well as depth. Some students like to aim for an arbitrary number of quotes per paragraph (which can work well, but also might get in the way of good analysis, so its up to you) but 'd recommend at least forcing yourself to impose a window of, say, between 8-14 quotes that you have to use, even if some are only quick, one word ones. It'll depend on the length of your paragraphs, but it will give you a clear goal to work towards, as well as some idea of whether you're over/underestimating the amount of proof you have.

Hope that helps; all the best with it!

Feedback for lauren7366
Stories of our past help us discover who we are in the present.
To what extent does this confirm your understanding that discoveries can be reassessed over time?


Discoveries can be a process of reassessment, which are catalyzed due to change of circumstance and reflection of on one's (apostrophe for possession) past mistakes. As a result of these confronting and provocative discoveries, enlightenment is reached in the reformation of our morals, encouraging humanist values such as forgiveness and love. V. good opening. William Shakespeare’s pastoral comedy, “The Tempest” written in 1610, and the film, “The count of Monte Cristo,” directed by Kevin Reynolds, both exemplify the corrupt desire for vengeance that allows for a renewed perception of world view once it is realized to be deceitful. I get the underlying idea here, but the sentence structure is a tad muddled; I'll dissect this in the end comments. This realization comes through reflection of the past in order to progress in future endeavors. Final sentence feels a little short and stilted, and it seems like there are some other facets to your argument that could be fleshed out here to add to this, but an otherwise good intro :)

The Tempest follows the reformation of an individual's (apostrophe) absolute control and power, which are transformed into benevolent traits such as internal judgment and empathetic characteristics that would thrive within utopian governance. This is achieved by self acceptance of past faults providing a renewed perception of compassion and forgiveness; notions that are the fundamentals of humanism. When remarking to Miranda the “foul play”, that they were “heaved thence,” Prospero highlights the corrupt society in which he ruled, where he himself “neglect[ed] worldly ends, all dedicated to the closeness and the bettering of [his] mind,” but also the world in which he was betrayed by “ a brother […] so perfidious.” Quote integration is perfect, but be careful not to bombard your reader with too much evidence at once. In the next sentence you state that Shakespeare utilises this but it took me a second read-through to work out what 'this' was because of just how much you'd mentioned here. It might be better to scatter your evidence across a few sentences so you can analyse things independently, then combine them for the big 'Therefore Shakespeare implies...' moment at the end. Shakespeare utilizes this as a microcosm for the power lust that humanity exemplifies in positions of authority; knowledge prized over our collective morality and familiar bonds. The 1600’s imperialistic context of control and influence over the majority, no comma needed here is prevalent within Prospero’s need to manipulate native Caliban and Ariel’s servitude for personalized advantage. V. good. This is showcased using the disruption of natural order, the tempest itself, symbolizing the brazen attitude that Prospero subjects’ no apostrophe needed here others to; shouldn't be a semicolon here either. A comma would suffice, though even that's optional as a result of his ultimate thirst for revenge that dominates all cognitive deliberation. <Linking word? (eg. 'Therefore...' 'Consequently...')>Audiences grapple with Prospero’s shortcomings to understand that life manipulates the circumstances that we oversee, however it is how we confront these circumstances that determines our future success. Good point, and I absolutely love that you're taking this a step further with your 'however...' statement. However (:P) you need to ensure that your final statement is still supported by the content of your paragraph. Here, you've justified the notion of life manipulating circumstances, but you haven't really tacked the idea of confronting said circumstances. Just be careful with these assertions, as some teachers will still give you credit for this whereas others will see it as moving too far beyond the scope of your previous discussion.

Individuals in powerful positions often have to experience unprompted failure in order to reassess corrupt core values and furthermore access personalized freedom and amend flawed leadership qualities. It is through Prospero’s relationship with spirit Ariel, that access to worldly insight is obtained; shouldn't be a semicolon here - see end comments re: punctuation from self absorption into acceptance of the altruistic experiences in life, taking from this a renewed optimism for the future. Prospero’s individual values of hatred hatred of what, exactly? There's an opportunity for more description here, or perhaps even evidence are challenged by Ariel’s perspective of tenderness and wisdom in the face of choice, “that if you now beheld them, your affections would become tender.” This is then furthered by the acknowledgment of the humanistic characteristics that Prospero should adhere to by reason because of his biological makeup, “mine would sir, if I were human.” this quote isn't integrated as well as all your others. From this, audiences are reminded of the humanity we are answerable word check. This doesn't really fit here to exhibit in order to thrive as a society, however Prospero also learns that his corruption is what erodes humanism's core belief; if we cannot have faith in the people, a strong fundament(???) of politics cannot be developed. We also see this realised in alliteration, “rarer act is in virtue than in vengeance”, symbolising the discovery of human connection and that kindness prevails over the corrupt quest of power. Yes, but how does that alliterative language symbolise that notion of discovery? Remember to spell out your evidence. Also, the ends of your paragraphs should be for zooming out, which you have done in the second half of this sentence, but bringing up new evidence here is a little risky, so try and cover that earlier to grant yourself time enough to talk about big ideas here.

Prospero’s epiphany similarly coincides with Edmond Dante’s experience whereby his integral belief in vengeance is challenged by Abbe Farria (priest) put this information into your sentence grammatically; don't just stick it in brackets and the revelation of a biological son, leading to a rewarding sense of love and peace from one’s if you're talking about specific characters, then you don't have to use the generalisable 'one'. That's just for the broader statements about discovery as a whole inner demons. Chief prosecutor Ville fort; no semicolon one who originally deceives Edmond into expression - you can't 'deceive someone into something' imprisonment, foreshadows the enlightenment that Edmond is to experience at the end of the film, “perhaps some good will come out of this treasonous affair” this isn't integrated. Audiences rather than making 'Audiences' the focus of your sentence, use 'The author + verbs' (see examples here if needed) understand this enlightenment to be that he discovers the intrinsic value of knowledge to be a tool, see end comments providing the wisdom to forgive even in the face of ultimate choice. The unpredictable discovery of his biological son, through exclamatory language accompanied by directed close up, “Albert, you are the son of Edmond Dantes. The man you know as the Count of Monte Cristo,” allows Edmond to reassess the importance of a personal vendetta when presented with opportunity to rebuild love ; a founding characteristic that is vital in order to further one’s humanism.
Okay, this para conclusion is quite close to the prompt, but the linking could be clearer. See below for fuller explanation of this.
Awesome structure, and some decent analysis to back it up, but I'm a little concerned about the relevance of this to the prompt. More specifically, I can see the relevance, but if I'm the assessor, I shouldn't be the one having to think 'how does this relate to the idea of past stories resulting in discoveries about present identities?' -- YOU'VE got to make those connections really explicit, and the less thinking your assessors have to do, the better.

Across your whole body paragraphs, you don't use the words 'past' or 'present' more than once, and they really need to be the central pivots of your discussion in this case.That's not to say you have to overuse the words to the point of repetition, but they should be employed fairly regularly to ensure that the assessors know what links you're trying to make.

Aside from that, the bulk of your discussion was pretty good. You could use a little bit more fluidity between your ideas in order to connect them, but the way you build your argument is on point - you just need to direct it in the right way by ensuring the links to the prompt are really overt.

There were also a few sentence structure issues like with:
"[The two texts] both exemplify the corrupt desire for vengeance that allows for a renewed perception of world view once it is realized to be deceitful."
or
"Audiences understand this enlightenment to be that he discovers the intrinsic value of knowledge to be a tool."
These problems seem to arise when you're using the passive construction, which is where instead of saying 'He discovers X to be Y' you say something like 'X is discovered by him to be Y.' It's perfectly valid, but can, in some contexts, sound a little clunky, especially if overused. I won't go into dissecting these sentences in too much detail, but suffice it to say that you're using passive constructions as well other complex grammatical features that are adding too many layers to your sentence to the point where it's obfuscating clarity. Take the second example - I could easily transform this into: 'Thus the author suggests that his understanding of knowledge to be an intrinsically valuable tool is an important enlightenment...' and it's much more direct.

So from here on in:

1) Keep up the great work with spelling out your evidence, but try and distribute this evidence roughly evenly across your body paragraphs, ensuring that you integrate the quotes when you do. Think of it like jam on toast. Technically, if you just lump a spoonful of jam on the corner of the bread, you've gotten the right amount of jam, but if you don't spread and dispense it properly, you're in for an unsatisfying breakfast experience.

2) RELEVANCE RELEVANCE RELEVANCE! I can see that you know how your ideas connect to the prompt, but I can't give you credit for what I'm straining to see, so bring those links to the surface and reinforce them in your topic sentences and paragraph conclusions.

3) Work on finding some complex connections between your two texts; I know you noted that you hadn't discussed your related text in too much detail just yet, but it really is essential later down the track. For now, don't worry about doing this in an essay format and just work with the ideas themselves. Maybe collate a bunch of points for each, assign them all numbers or colours, and just create a visual display of how they might relate together. There'll likely be some points that don't have direct parallels or contrasts in the other text, but that's fine too. Not every point will have to be compared, though you will need a substantial amount, and it'll help you feel much more prepared for future assessment.

4) Watch out for your sentence structures, and stop yourself occasionally to ask 'is there a better, simpler way I could put this?' If in doubt, simplify, and just prioritise getting your point across clearly and unambiguously.

5) Careful with punctuation! It's a tiny thing, and some of these might just have been typos, but it irks assessors, and it's usually a very easy fix. In my experience, students can go from not knowing what a semicolon even looks like, to using them confidently in under an hour. Just look up some explanations and sample sentences online, and you should be fine with it :)

Feedback for foodmood16
'Discoveries can be transformative and/or far-reaching for an individual'

The emotional, intellectual and physical discoveries can act as the foundations for an individual what do you mean by this? Foundations for what? Their identity? Their lives? Their sense of self? I know the prompt is quite broad, but your answer should be specific! and can have a transformative effect on their awareness of human experiences and the wider world. The discoveries can be carefully planned, so much that the composer word check - 'composer' is usually used to refer to authors of musical pieces, and even then only really for classical pieces (e.g. Mozart) can place an individual in unauthentic ??? situations, even without their knowledge. In the 2011 documentary series ‘Go Back To Where You Came From’, the participants are taken on the migrant journey in reverse and the experiences of Adam Hartup and Darren Hassan show many emotional ramifications that are impacted by their awareness of the influence of the media on this political issue. The producers have seemingly composed many of the experiences to exaggerate the extent of their discoveries, this feels a bit like you're evaluating the show, saying that they exaggerated things or made them up in order to get their point across, and this isn't really your job in this piece. Try to reserve judgement and just comment on the meaning being created making the responders question the far-reaching impact of the emotional responses of the participants. Similarly, in the 1963 poem ‘Mid-Term Break’ by Seamus Heaney explores the transformative careful with sentence structure - read this again and you'll likely see the issue ('in the poem by the author explores...') discoveries of his own childhood, having lost an infant in a traumatic accident. He focuses on how this impacted the people around the protagonist, as well as the reader. He uses carefully planned literary techniques in order to provoke an emotional response from the reader. Of course he does! That's what poets do. Sentences like these, accurate though they may be, are often not given much credence because they're too generic. It'd be like saying 'The author of this novel has explored a vast array of complex and multifaceted ideas through the use of complex literary devices and structural features.' Sounds nice, but I've said nothing in that sentence other than 'there are things in this novel that are explored in a novel-y way' :P Both texts explore how discoveries can be influenced by the composer by making the audience As much as I like that you're focusing on both the characters and the audience, you seem to be dodging the prompt a little bit. Your task is not to comment on how the texts might manipulate the audience to help them discover things, but instead to examine how the authors/creators present the idea of discoveries being transformative for those in the texts (i.e. the people on 'Go Back' and the speakers/personas in the poetry.) and protagonist aware or unaware of these influences. This seems like an odd, slightly fence-sitting statement - the author either makes them aware or unaware - is there not something more definitive you could say other than 'it's either one or the other?'

The foundations of transformative discoveries can be affected by the authorial presence and careful planning of a reality TV series, and ultimately the participants are unaware of this presence You're zooming into the text too quickly here. Try and make a broader statement about discovery and your sub-argument first, and then bring up the idea of authority, reality tv, and the show itself. It's like if you said 'The nature of complicated discoveries is often caused by Adam when he says "..." ' - you can't go from a zoomed-out comment on something abstract all the way into the text too soon, or it's jarring for your reader. During Episode 2 of ‘Go Back To Where You Came From’, the participants take part in a police raid on a construction site. On the ride to the site, Adam comments, “if it’s the Chins haven't seen this particular episode, but what does this mean? Why is this quote significant? I’ll lose my shit… I won’t be able to handle it.” The non-diegetic music builds into a suspenseful tone adds to the suspense and the camera shows Adam in a close-up shot. This shows Adam’s unawareness of how his reaction to this confronting event can be manipulated by the producers. How does it show this?? The emphatic language what language? Are you talking about the quote from three sentences ago? What language in particular was emphatic? And what was it emphasising? shows the growing attachment and understanding Adam has developed through the emotional discoveries. Throughout the raid, Adam doesn’t say much, but his body language expresses his confusion and confrontation Firstly, how does his body language show this? What does 'expressing confusion' look like in this context? BE SPECIFIC! Secondly, if you reverse the order of these two words, you'll see that confrontation doesn't really belolng here: 'his body language expresses his confrontation and confusion' - see how 'his confrontation' sounds a little odd?, unaware of how to act in retaliation to the site of arrested refugees. As new discoveries are made, it becomes clear to the participants and audience how an authorial presence can shape material into their preferred ideal. So are you arguing that Adam wasn't actually discovering anything and that the show's creators were just 'shaping the material?' I'm a bit confused as to what you're trying to argue here - you start by saying 'As discoveries are made...' but what discovery are you talking about? That should be your focus here, not the notion of 'authorial presence.'

For the reader, transformative discoveries and their foundations are affected by the author of a text through their careful use of literary techniques, with which allows the reader to emotionally respond to the text you're still zooming in a bit too quickly here. Focus on discovery and the prompt for at least a sentence before you do this. In the poem, ‘Mid-Term Break’, many techniques are used to convey the story of the death of an infant, although the reader doesn’t discover this until the end of the journey. The title itself “Mid-Term Break”” leads the responder to think the poem is about a happy event of the end of the school year, making them unaware of the poignant and traumatic story that lay ahead. It is the end of the first stanza that the reader suspects that something unusual is about to occur, though still oblivious to the traumatic event. “At two o’clock our neighbours drove me home.” This unusual event of the neighbours taking the protagonist home allows the reader to prepare themselves for the death of the four-year-old boy this is a bit of a leap :/ The step-by-step logic of your discussion is excellent, but this one is quite a stretch - the neighbours drive a kid home, which prepares readers for the death of a 4 year old?, building the morbid mood. They are still unaware of the emotional consequences of the death. You're resting this whole paragraph on one point, which is quite risky. What about all those 'many techniques' you talked about earlier? You could use a bit more evidence in this paragraph. Far-reaching discoveries can be impacted by the author’s manipulation of words examples? to lead to greater the impact of the responder’s emotional and what emotions would these be, exactly? What kind of response is being evoked? And most importantly, how is this transformative and/or far-reaching? response.

As new understandings and perceptions develop from discoveries, it becomes clear how constructed the media is in order to portray a particular side of a story. The producers of ‘Go Back To Where You Came From’ have aimed to emphasise their pro-humanitarian motivation to provoke an empathetic response by the audience again, I feel like you should be focusing on the nature of the people in the show's discoveries about the world and the suffering of refugees, rather than subtly criticising the show and saying that they're manipulating the facts for their own agenda or anything like that. At the end of Episode 1, the participants are placed on refugee boat, and are forced to travel into the open sea without knowledge of their destination. After experiencing this unauthentic how is it 'unauthentic?' Surely this is one of the most realistic moments since when refugees seek asylum, the process of travelling is often risky and uncertain, especially if they're paying some dodgy people smuggler - they have no way of knowing for sure where they'll end up either situation, the live camera captures Darren, the most opposed to boat people, express his concerns that the media is “…emotionally involving us without our consent.” The collective terms used in his language shows that he is talking not only about himself, but the Australian public. Darren’s body language and tone describe these in more detail - what is it about these things that betray his agitation also shows his agitation by the experience, and shows his awareness of how the program has constructed these ‘transformative’ experiences in order to provoke an emotional response by the participants. He also explains that the media wants “you should feel bad, you should feel empathy.” Darren is placed in a mid-shot, and uses high modality language why is this important? How does it highlight antything? to highlight his feeling of manipulation by the media. The awareness of constructed discoveries that an individual may undertake can therefore influence their shift in perceptions, or even affirm their previous understandings. Good, but you need to focus more on Darren. What happens to him throughout the show, for instance? Does he change his mind, or is he completely cynical about the emotional manipulation that he thinks is taking place? Do his perceptions shift at all?

The new understandings and renewed perceptions that derive from the foundations of discoveries are impacted by the author’s choice of form and stylistic elements of their writing. In the poem ‘Mid-Term Break’, Heaney explores the parents' apostrophe after the s if it's plural and possessive reactions to the death of the infant, before the protagonist’s is discovered. In stanza 2, the father’s reaction is distinguished word check. “… I met my father crying…always taken funerals in his stride…” This quote isn't integrated at all; try and fit this into one of your sentences. In the 1950’s, the thought of a man crying was against the society’s stereotypical man, and the use of this emphasises the lasting impact of the death of a loved one, especially an infant.  The mother’s reaction to this terrible news is expressed in stanza 5. “…coughed out angry tearless sighs.” see above The silence of “tearless sighs” accentuates the idea of the reverence of death. At this stage, it is still unclear to the audience of who is dead, though the characters portrayed have already discovered this.<-- I'm not seeing the link between that previous statement and this one --> The awareness of the characters show how discoveries build to the transformative foundations of the audience and their new understandings. What discovery? What transformation? What foundations? What new understanding?

In conclusion, both the SBS documentary series ‘Go Back To Where You Came From’ and the poem ‘Mid-Term Break’ explore how the audience and participants are aware or unaware of how the composer influences the discoveries they make. This conclusion is a bit of a weak spot as you're just restating a line from your introduction without much development. You could do a lot more here in terms of linking this to the prompt and saying something about the nature of discovery overall, which should ultimately be your focus here.
I think there are some inconsistencies with your approach in this piece, because there were sections where your analysis was highly accurate and impressive, but others where you seemed to veer off course and talk about the manipulation of authors, particularly in your paragraphs dealing with 'Go Back.' Perhaps it was just a one-time issue that came about because of the breadth of the prompt, but you need to ensure your sub-arguments are always on track.

If you're talking about discoveries being transformative and far-reaching, then that's the concept you should be exploring in the text. Don't think about how the author might be 'faking' these ideas, or whether the audience are the ones making the discoveries - you have to analyse what the author does and explain how textual evidence supports those points.

In other words 'what does this show tell us about discovery?' Or, more applicably for this essay 'How does 'Go Back' + Heaney's poem show us that discovery can be transformative and far-reaching?' THAT'S the question that needs answering here.

Which leads me on to my next point - you don't really ever explain what 'transformative' means, and you hardly touch on the whole 'far-reaching' thing at all. Those key words in the prompt should be the crux of your piece, and you need to prioritise these.

Your essay structure was pretty good, though the starts and ends of your paragraphs could use a bit of tightening up. Your topic sentences were a little bit too specific to the texts, and I think you'd benefit from a more abstract point to start off with before then linking this idea to whichever text you wanted to look at. Also, (and again, there's more advice on this in previous feedback posts) there isn't much of a link between your two texts because you're only ever looking at them in isolation. Thus, instead of going for one alternative paragraphs on each, you could instead try:

Paragraph 1: 75% focus on 'Go Back' + 25% on a related idea in Heaney.
Paragraph 2: 50% on 'Go Back' + 50% on a related idea in Heaney
Paragraph 3: 75% on Heaney + 25% on a related idea in 'Go Back'

or something like that. Basically, find a concept like 'discovering new things can transform us for the better,' and link that to one of the texts. For instance, some of the participants in 'Go Back' gained a more enlightened and more informed view of the refugee experience, resulting in a more empathetic stance on related political issues. Then, we can connect this to the second text by either finding a point of similarity or difference. If I wanted to find a point of difference here, then I might say that in Heaney's poem, the parents discover something that will inevitably change their lives, and have overtly negative ramifications on their relationship and their mental state. Thus (getting to the end of the paragraph now) we can conclude that although certain discoveries can be very advantageous for one's psyche, they can also involve a great deal of suffering.

Then you move on to the next idea, finding points of similarity and difference as you go.

In short:

1) Know what the task is asking of you, and keep that in mind while writing. If it helps, write out a series of questions that you need to answer per paragraph so that if you ever do lose focus or forget where you're going, you've got that framework there as a reference point.

2) Forge links between your set texts so that you're able to use both of them in order to say something about discovery. It's kind of like your goal is to paint something purple, and you have a tine of blue paint and a tin of red paint. At the moment, you've painted half the thing blue and half the thing red... so your approach is theoretically good and you're using the right materials, but ultimately it's not going to result in the purple that we're after. Only by combining your ideas through comparison and contrast can you get that desired effect.

3) Make sure the first and last line of your body paragraphs are just about discovery, not about the texts. This isn't a hard-and-fast rule, and if you read some of the other essays in this thread, you'll likely notice they can disobey this but still do things well. For now though, if you can impose that restriction on yourself, you'll be able to better adjust to the zooming in and out process that's necessary for success.

4) Give yourself enough ammunition throughout your essay so that by the time you get to your conclusion, you're confident enough to shoot your target. If you haven't build up a good range of sub-arguments, it can be very difficult to do all that work in the final few sentences, so each time you conclude a paragraph, take things back to the prompt and question how what you've been discussing pertains to the prompt's focus. Then, answer that question directly and unambiguously so that the assessors will have no choice but to give you marks for relevance.

5) Sentence structure and vocab are mostly good, though there are a few instances where your word choices let you down a bit. As I've said before though, the more risks you take now means the more opportunities you have to correct your mistakes, so keep experimenting with expression in order to further enhance your writing.

Best of luck!

brenden

  • Honorary Moderator
  • Great Wonder of ATAR Notes
  • *******
  • Posts: 7185
Re: English Advanced Essay Marking (Modules Only)
« Reply #72 on: January 30, 2016, 02:12:59 pm »
Comments below; let me know if you guys have any questions! :)

Feedback for ssarahj
Really good stuff here! You seem to have the underlying structure well under control, and the clarity of your expressions was pretty much flawless.

There's some improvement to be made with regards to 'spelling out' your evidence and ensuring the assessors can get from A (your quotes and textual examples) to B (your overall ideas).

Taking this excerpt from your first paragraph:
"She describes her pain as being, “everywhere, isn’t it? In the air we breathe”, in which Gow uses a rhetorical question and a metaphor to demonstrate the all-consuming and hopeless nature of her grief."
and breaking this down into its essential bits, we have:
- the quote: "[pain is] everywhere, isn't it? In the air we breathe"
- the devices: (rhetorical question & metaphor)
- the meaning: Coral's grief is all-consuming and hopeless
but how are these pieced together? You state that there is a connection (and there totally is!) but if you were to make this explicit, your assertion would be a lot stronger. So, in this case, how do you know her grief is all-consuming and hopeless? How does that quote demonstrate your point?
See this for a possible answer
If we're to focus on the metaphor part of the quote, we've got to try and get from "[pain is] in the air we breathe" to the notion of hopelessness and consumption, so what is it about this language that might lead us to this idea? When, the concept of something being "in the air we breathe" implies that it is not only all around us (=all-consuming, or at least ubiquitous) but that it also infiltrates us and becomes a part of the air. Thus, to breathe (and to live) is to experience pain, because it is so ubiquitous an unavoidable. Now we can reasonably conclude that there is an element of saddness and hopelessness in that quote because of this rationale, so your analysis might look something like:
She describes her pain as being "everywhere, isn't it? In the air we breathe" whereby Gow's metaphorical description of pain being such an omnipresent and inescapable force serves to demonstrate the all-consuming and hopeless nature of Coral's grief.
It may seem like a small change, but that one tiny step of 'showing your workings' can be very advantageous in showcasing your knowledge of the text and ability to extract ideas from it.
Also, and I believe I may have mentioned this in a previous bit of essay feedback but just in case: you want your paragraphs to build on one another. If I were to change the order around and read your 3rd para first and your second one last or w/e, it should seem weird! You don't want your essay to be an assortment of random mini-discussions that go off on their different tangents without linking together; instead, you want it to be like a pyramid with each brick aiding you in reaching that high point of your thesis statement in response to the prompt. You still have to have separate paragraphs, of course, but you should see them as a series of stepping stones to get you to an overall point, rather than being like... I don't know... flower petals that just stem out from the centre but don't combine help you get anywhere with your arguments.

Beyond that, some greater variation in sentence structure and vocabulary would also help you bolster your piece, especially when it comes to the conclusion. You always want to end with impact, as this is where the mark is decided, so to restate your points and only use sentences that have basically already been used earlier is to tell the assessor you don't have anything better to say, meaning that if they're hovering between a 7/10 and an 8/10, a mediocre conclusion might be what tips the balance to a 7 rather than an 8.

Also, and this might be a matter of some contention depending on your teacher, but I'd say it'd be great to have a paragraph (or multiple paragraphs, ideally) that talk about both of your texts instead of switching between them both. If you are to forge connections between the two, you should really talk about them together. See some of the previous corrections for advice on potential essay structures.

But you're already in the enviable position of basically having all the content right, and just needing to fine tune your presentation of it. So, the stuff you should aim to work on:

1) Have your ideas and sub-arguments build on one another. This is often best practised through essay plans as opposed to full essays, so maybe draw up some rough outlines and give yourself some idea of how things will link together to help develop your thesis statement.

2) Ensure that you're connecting your examples to your ideas as directly as you can, with the emphasis on explaining how certain evidence demonstrates your point.

3) Vary your sentence structures, and try to avoid taking your body paragraph topic sentences and just listing or rewording them in the intro, B.P. concluding sentences, and conclusion.

Very good effort overall :)

Feedback for MemeKing
Very interesting discussion with some really complex ideas at play here, but you need to be careful not to let good words get in the way of great ideas, so to speak. There were times where your arguments were a little unclear of confusing, and your vocabulary is kind of exacerbating that issue rather than masking it.

In English, vocab is like an amplifier. If you're doing things well (like by spelling out your evidence and building up relevant ideas) then your vocabulary will usually magnify that good stuff and make it even better. But if there are problems with your understanding with the text (which doesn't seem to be an issue for you) or with the clarity of your ideas (- more pertinent, in this case -) then repetitious sentence structures or out-of-place words can draw attention to those concerns.

As such, I'd recommend prioritising clarity, even if it's at the expense of sophistication. It doesn't seem like you'll have any problem transforming your ideas into complex, well-worded sentences once you've sorted them out, but it's that sorting out that needs to happen first! Otherwise, it's like you're adding icing and cake decorations to a cake that hasn't been baked yet :P

I'm also a little bit confused as to the structure of your 'essay.' Maybe it's because I'm just a humble VCE-er and I don't know how your school operates, but unless this was a kind of short answer response, it seems very odd that you wouldn't be required to have a formal introduction and conclusion with three to five body paragraphs, as I know that's what the end-of-year standard tends to be. Perhaps this was a different exercise, but if you were asked to write an essay and you instead wrote two somewhat disconnected paragraphs that conveyed your ideas, that might've contributed to you're not doing too well in the last assessment.

Having said that, the structure of those paragraphs was really good; you've got the abstract unpacking of your ideas at the start, and the zoomed out summative sentences at the end, which gives things a nice, compact, and most importantly precise focus. The only thing that needs work in that regard is your linking between evidence and ideas. I've written about this quite a bit in previous essay comments, so perhaps go back and read those if you need further clarification, but in short, you need to explain how the evidence you're using supports your points. Sometimes you can get away with not doing this (i.e. not every sentence of your body paragraph has to be an incredibly detailed, lengthy discussion of why X demonstrates Y,) but it is a surefire way to impress the assessor and boost you into higher territory.

I'd also say you'd need more evidence in order to make your ideas that much stronger. Your use of quotes and metalanguage were solid, but you just need to do it more often. Both paragraphs here had one moment towards the end of really excellent analysis, and you want to be replicating that kind of quality three or four times per paragraph.

Things to work on:

1) Ensure you can explore the connections between evidence and argument as clearly as possible, and that your expression doesn't get in the way of this.

2) Keep experimenting with vocab and using new words, as this is the only way to truly acquire a better vocabulary, but look up definitions and synonyms if you're unsure, as you have the opportunity to use such resources now even though you won't in assessment tasks. I know I've isolated a couple of word checks, but honestly, I'd rather you made a hundred mistakes now and learned from each of them than made none at all and never developed your arsenal, so keep that up because your vocabulary is clearly well-developed and will be a huge strength this year if you can iron out the little things.

3) Make sure you have a substantial basis of evidence for making your claims. This goes both for the small level sentence-by-sentence assertions in that if you don't back up 90% of what you say, it's very hard to attain marks, but also on the broader paragraph-by-paragraph level in that you need breadth as well as depth. Some students like to aim for an arbitrary number of quotes per paragraph (which can work well, but also might get in the way of good analysis, so its up to you) but 'd recommend at least forcing yourself to impose a window of, say, between 8-14 quotes that you have to use, even if some are only quick, one word ones. It'll depend on the length of your paragraphs, but it will give you a clear goal to work towards, as well as some idea of whether you're over/underestimating the amount of proof you have.

Hope that helps; all the best with it!

Feedback for lauren7366
Awesome structure, and some decent analysis to back it up, but I'm a little concerned about the relevance of this to the prompt. More specifically, I can see the relevance, but if I'm the assessor, I shouldn't be the one having to think 'how does this relate to the idea of past stories resulting in discoveries about present identities?' -- YOU'VE got to make those connections really explicit, and the less thinking your assessors have to do, the better.

Across your whole body paragraphs, you don't use the words 'past' or 'present' more than once, and they really need to be the central pivots of your discussion in this case.That's not to say you have to overuse the words to the point of repetition, but they should be employed fairly regularly to ensure that the assessors know what links you're trying to make.

Aside from that, the bulk of your discussion was pretty good. You could use a little bit more fluidity between your ideas in order to connect them, but the way you build your argument is on point - you just need to direct it in the right way by ensuring the links to the prompt are really overt.

There were also a few sentence structure issues like with:
"[The two texts] both exemplify the corrupt desire for vengeance that allows for a renewed perception of world view once it is realized to be deceitful."
or
"Audiences understand this enlightenment to be that he discovers the intrinsic value of knowledge to be a tool."
These problems seem to arise when you're using the passive construction, which is where instead of saying 'He discovers X to be Y' you say something like 'X is discovered by him to be Y.' It's perfectly valid, but can, in some contexts, sound a little clunky, especially if overused. I won't go into dissecting these sentences in too much detail, but suffice it to say that you're using passive constructions as well other complex grammatical features that are adding too many layers to your sentence to the point where it's obfuscating clarity. Take the second example - I could easily transform this into: 'Thus the author suggests that his understanding of knowledge to be an intrinsically valuable tool is an important enlightenment...' and it's much more direct.

So from here on in:

1) Keep up the great work with spelling out your evidence, but try and distribute this evidence roughly evenly across your body paragraphs, ensuring that you integrate the quotes when you do. Think of it like jam on toast. Technically, if you just lump a spoonful of jam on the corner of the bread, you've gotten the right amount of jam, but if you don't spread and dispense it properly, you're in for an unsatisfying breakfast experience.

2) RELEVANCE RELEVANCE RELEVANCE! I can see that you know how your ideas connect to the prompt, but I can't give you credit for what I'm straining to see, so bring those links to the surface and reinforce them in your topic sentences and paragraph conclusions.

3) Work on finding some complex connections between your two texts; I know you noted that you hadn't discussed your related text in too much detail just yet, but it really is essential later down the track. For now, don't worry about doing this in an essay format and just work with the ideas themselves. Maybe collate a bunch of points for each, assign them all numbers or colours, and just create a visual display of how they might relate together. There'll likely be some points that don't have direct parallels or contrasts in the other text, but that's fine too. Not every point will have to be compared, though you will need a substantial amount, and it'll help you feel much more prepared for future assessment.

4) Watch out for your sentence structures, and stop yourself occasionally to ask 'is there a better, simpler way I could put this?' If in doubt, simplify, and just prioritise getting your point across clearly and unambiguously.

5) Careful with punctuation! It's a tiny thing, and some of these might just have been typos, but it irks assessors, and it's usually a very easy fix. In my experience, students can go from not knowing what a semicolon even looks like, to using them confidently in under an hour. Just look up some explanations and sample sentences online, and you should be fine with it :)

Feedback for foodmood16
I think there are some inconsistencies with your approach in this piece, because there were sections where your analysis was highly accurate and impressive, but others where you seemed to veer off course and talk about the manipulation of authors, particularly in your paragraphs dealing with 'Go Back.' Perhaps it was just a one-time issue that came about because of the breadth of the prompt, but you need to ensure your sub-arguments are always on track.

If you're talking about discoveries being transformative and far-reaching, then that's the concept you should be exploring in the text. Don't think about how the author might be 'faking' these ideas, or whether the audience are the ones making the discoveries - you have to analyse what the author does and explain how textual evidence supports those points.

In other words 'what does this show tell us about discovery?' Or, more applicably for this essay 'How does 'Go Back' + Heaney's poem show us that discovery can be transformative and far-reaching?' THAT'S the question that needs answering here.

Which leads me on to my next point - you don't really ever explain what 'transformative' means, and you hardly touch on the whole 'far-reaching' thing at all. Those key words in the prompt should be the crux of your piece, and you need to prioritise these.

Your essay structure was pretty good, though the starts and ends of your paragraphs could use a bit of tightening up. Your topic sentences were a little bit too specific to the texts, and I think you'd benefit from a more abstract point to start off with before then linking this idea to whichever text you wanted to look at. Also, (and again, there's more advice on this in previous feedback posts) there isn't much of a link between your two texts because you're only ever looking at them in isolation. Thus, instead of going for one alternative paragraphs on each, you could instead try:

Paragraph 1: 75% focus on 'Go Back' + 25% on a related idea in Heaney.
Paragraph 2: 50% on 'Go Back' + 50% on a related idea in Heaney
Paragraph 3: 75% on Heaney + 25% on a related idea in 'Go Back'

or something like that. Basically, find a concept like 'discovering new things can transform us for the better,' and link that to one of the texts. For instance, some of the participants in 'Go Back' gained a more enlightened and more informed view of the refugee experience, resulting in a more empathetic stance on related political issues. Then, we can connect this to the second text by either finding a point of similarity or difference. If I wanted to find a point of difference here, then I might say that in Heaney's poem, the parents discover something that will inevitably change their lives, and have overtly negative ramifications on their relationship and their mental state. Thus (getting to the end of the paragraph now) we can conclude that although certain discoveries can be very advantageous for one's psyche, they can also involve a great deal of suffering.

Then you move on to the next idea, finding points of similarity and difference as you go.

In short:

1) Know what the task is asking of you, and keep that in mind while writing. If it helps, write out a series of questions that you need to answer per paragraph so that if you ever do lose focus or forget where you're going, you've got that framework there as a reference point.

2) Forge links between your set texts so that you're able to use both of them in order to say something about discovery. It's kind of like your goal is to paint something purple, and you have a tine of blue paint and a tin of red paint. At the moment, you've painted half the thing blue and half the thing red... so your approach is theoretically good and you're using the right materials, but ultimately it's not going to result in the purple that we're after. Only by combining your ideas through comparison and contrast can you get that desired effect.

3) Make sure the first and last line of your body paragraphs are just about discovery, not about the texts. This isn't a hard-and-fast rule, and if you read some of the other essays in this thread, you'll likely notice they can disobey this but still do things well. For now though, if you can impose that restriction on yourself, you'll be able to better adjust to the zooming in and out process that's necessary for success.

4) Give yourself enough ammunition throughout your essay so that by the time you get to your conclusion, you're confident enough to shoot your target. If you haven't build up a good range of sub-arguments, it can be very difficult to do all that work in the final few sentences, so each time you conclude a paragraph, take things back to the prompt and question how what you've been discussing pertains to the prompt's focus. Then, answer that question directly and unambiguously so that the assessors will have no choice but to give you marks for relevance.

5) Sentence structure and vocab are mostly good, though there are a few instances where your word choices let you down a bit. As I've said before though, the more risks you take now means the more opportunities you have to correct your mistakes, so keep experimenting with expression in order to further enhance your writing.

Best of luck!
You are literally unbelievable.
✌️just do what makes you happy ✌️

MemeKing

  • Adventurer
  • *
  • Posts: 6
  • School: James Ruse
  • School Grad Year: 2016
Re: English Advanced Essay Marking (Modules Only)
« Reply #73 on: January 31, 2016, 06:12:30 pm »
Hey thanks for the feedback, should've mentioned I didn't include my introduction and conclusion!  I tried changing some things up and attached is the intro + both body paragraphs - Thank you!

Discovering the duplicity of truth in art allows for the appreciation of previously lost value and ultimately rekindles the ability to appreciate art through time.  Simon Nasht in Frank Hurley examines that the mutability of art can only appreciated once we discover and accept its dualistic nature.  Similarly, Nasht atones to conveying the human condition that provokes the need to preserve oneself through the catalyst of art.

The questioning of truth leads to an appreciation of value once we discover and accept its dualistic nature.  Nasht attempts to convince a now pluralist society that by accepting the duplicity of Hurley’s artworks can these “elaborate concoctions” be valued more than “outright fakes”.  Nasht initially portrays Hurley as a “conjurer” with a camera and through cinema verite establishes how Hurley “undermined their historical value”.  Nasht does this to critic the epoch of Hurley’s work through their inability to see value in the duplicity of his “manipulations”.  However, Nasht demonstrates the mutability of Hurley’s work through the juxtaposition between Hurley’s photos and their modern recreations once he turned “the battlefield into a canvas of his own making”.  Hurley’s artworks are split-screened by Nasht in order to contrast their similarities in the hopes that their ephemeral existence may be immortalised. The commemoration of Hurley’s artworks in a now pluralist society is now seen through the close-up commentary shot that “today composites would seem commonplace”.  Furthermore, Nasht’s embellishment of Hurley now coincides with the postmodern pluralist disposition of “a world searching for heroes” and furnishes a now accepting society the ability to question historical truth.  Ultimately, Nasht demonstrates that only through the questioning truth and accepting its dualistic nature can a more heightened discovery be formed.

The transcendence of art attempts to immortalize the otherwise ephemeral existence of ones legacy upon discovering their limitations.  In ‘Frank Hurley’, Hurley discovers he cannot perpetuate himself as a myth as he then attempts to immortalize himself through his work.  Hurley “saw a market for exciting adventure films”, and through leaving himself as being a “mere observer,” he attempts to promote himself as a “fearless photographer”.  Nasht’s diegetic sound of Hurley holding spears and skulls attempt to portray Hurley’s “glorious” existence that he attempts to preserve through his photographs.  Nasht furthermore highlights his ambition as although “New York was unimpressed”, the “showman hit upon a new angle”.  Hurley then juxtaposes his earlier work in Papua New Guinea with “the lost tribes of Israel” that “flamed up front page headlines” that ultimately brought attention to Hurley, albeit controversial.   Hurley used this fame and his discovery for “his love for drama” to create films that were intended to immortalize his work.  Although Hurley “made up quite a few stories”, Nasht ultimately edits the shot of the auction house in order to commemorate Hurley’s transcendence past a “mere photographer”.  Ultimately, Nasht portrays Hurley’s attempts towards preserving his legacy through the continual pursuit of his artworks. 


polpark

  • Adventurer
  • *
  • Posts: 5
  • School: RSC
  • School Grad Year: 2016
Re: English Advanced Essay Marking (Modules Only)
« Reply #74 on: February 06, 2016, 03:56:28 pm »
HI!!
This is my intro and first body to my A.O.S study essay!!
PLEASE give me lots of criticism since English is one of my weak subjects and I need to improve!!!
THANKS FOR MARKING OUR ESSAYS!!!! :D :D

“Discoveries are often evoked by curiosity and wonder, offering up new understanding of ourselves and the world we live in.” Discuss this statement in relation to your prescribed and one other text of your choice.

Regardless of their motivation, discoveries shape and redefine an individual’s identity, successively shifting their previous perspectives of the world and it’s controversial issues. Ivan O’mahoneys’s Go Back To Where You Came From, Suzzane Buffam’s “The New Experience” and J.K Rowling’s The Fringe Benefits of Failure, and the Importance of Imagination exemplifies the process of attaining awareness of what was once misunderstood or concealed. Through utilising techniques unique to their textual medium, the three texts explore various concepts of discovery, facilitating the close analysis of the results of discoveries evoked from varying catalysts.

Go Back To Where You Came From illustrates the transformation of individuals resulting from tangible experiences instigated through extraneous motives. In the introduction, the utilisation of cross-cutting between archival footage of a refugee boat crashing into the shore and the interview of politicians stating “we must stop the boats,” engender sympathy towards the refugees and thereby evokes antagonism toward the hostility of the politicians. The biased editing indicates that the catalyst of the refugee process the six participants embark on was the producers’ external motives to persuade the public’s opinion on the refugee issue. As the six participants proceed through their journey, the development of their physical discovery is portrayed through the scene in which Adam, Glenny and Darren travel through the red-zone. The U.S soldiers assurance of having a “SOP(standard operating procedure) in place to make sure they(bombers) don't get too close,” is capitalised on through the employment of close up shots of nearby vehicles. The close up shots of the vehicles conveys the anxiety and fear of the scene through raising suspicion of a possible bomber. This trepidation is further enhanced by the voiceover of the narrator “only an hour earlier a bomb exploded nearby, killing two civilians”. The participants’ response to this physical confrontation is captured through reaction shots that reveal the apprehension and fear brought by their tangible firsthand experience of the dangers the refugees flee from. Adam Hartup expresses the resultant change he has undergone due to the discovery as he states “I won’t say it’s illegal (entering Australia by boat), its too harsh of a title,” contrasting from his past accusation of refugees to be “criminals.” Adam’s quotes portray his realisation that categorising people who are simply attempting to escape death as murderers and thieves is not appropriate. The portrayal of Adam’s transformation of his perspective on refugees illuminates the effect of discoveries evoked extraneous to themselves.
« Last Edit: February 06, 2016, 04:00:26 pm by polpark »